GI 2

¡Supera tus tareas y exámenes ahora con Quizwiz!

hepatitis B

DNA virus! blood transmission YES chronicity via the Dane particle Causes acute & chronic disease you see "Ground glass Hepatocytes" prevention: pre/post exposure immunization, post exposure IG SEE CHART

while in the hepatocyte, the bile acids are conjugates to certain amino acids___

glycine or taurine this reduces the pKa and makes them more water soluble and better working pH of the duodenum

as pH goes down the amount of HCO3- secretion ___

goes way up! inversely proportional

Iodine deficiency

goiter, hypothyroidism

what is inversely regulated with cl in pancreatic duct?

hco3-

what is the pancreatic fluid secretion made of?

heavy concentration of bicarbonate in order to raise the pH of the chyme that is being dumped into the duodenum

Short chain fatty acids passively leave the cell and enter the _____

hepatic portal vein

where are bile acids secreted from?

hepatocytes

Salivary glands are a bit unique in that they are totally regulated by ___alone. and it is ___

nerves There's only neural regulation of salivary secretion. It's autonomic, both sympathetic and parasympathetic. Both branches of the autonomic nervous system innervate the salivary glands.

what are the major organic components of saliva?

proline-rich protiens, mucin glycoproteins, alpha amylase, lipase, RNase, Kalikrein, lactoperoxidase, lactoferrin, lysozyme, and IgA

during the instestinal phase, the neural regulation of pancreatic secretion is by:

vagovagaal reflex from afferents in duodenal mucosa responding to products of digestion

the luminal membranes of the salivary cells are relatively impermeable to__

water

pancreatic fluid is mostly made of:

water and NaHCO3

what is steatosis?

a Degenerative Change: Cell swelling (ballooning or feathery degeneration) Steatosis (accumulation of triglyceride) Accumulation of substances (i.e. iron, copper)

the fluid component of the bile works just like ___

a pancreatic duct cell

what are the organic components of bile?

bile acids, fatty acids, bilirubin, phospholipids, cholesterol, and proteins

why do you need bicarbonate in the bile fluid?

bile is an acid= bicarb is the buffer

what are the main functions of saliva?

lubrication, dissolves solid substances, starts the process of digestion, bactericidal, maintains the pH (is alkaline), temperature buffer,

what are the 2 types of diarrheas?

malabsorpative/osmotic and secretory

what are the product from amylase?

maltose (2 glucoses) maltotriose( 3 glucoses) or a limit dextran

the swallowing reflex center is located in the ___ of the brain

medulla

what is MMC?

migrating myoelectric complex -Stimulated by motilin -Contractions begin & proceed for a few feet and then start up again slightly further down the intestine -This is repeated until the MMC reaches terminal ileum & then a new complex is initiated in the stomach -MMCs begin approx. 3 hours after last meal & repeat until ingestion of food

what is required to be present for bile to enter he duodenum?

CCK

what is Achalasia ? also known as megaesophagus

"A Failure to Relax" LES has impaired relaxation, often combined with defect in esophageal peristalsis Results in functional obstruction associated with dysphagia, regurgitation and if protracted, weight loss May be due to auto-immune induced selective degeneration of non-adrenergic, non-cholinergic (Nanc: VIP/NO) inhibitory neurons; secondary achalasia also seen in Chagas' disease (T. cruzi infection) Treatment: surgery (cutting muscle), mechanical dilation, drugs (NO donors, nifedipine, botox)

in the pancreatic duct, the Cl-/Hco3- has 2 helpers to make more HCO3- go into the duodenum to buffer the acid from the stomach

CFTR cells and Na/ HCO3- cotransporter cells

what occurs in traveler's diarrhea?

- It works in two ways: one by producing a toxin and a toxin that can activate cGMP which can activate PKG which can activate CFTR. Partly, travelers diarrheas work by activating CFTR, but they are more sneaky than that. They work by activating CFTR and inhibiting the sodium proton exchanger. The toxin is called STa. It was a heat stable toxin. It inhibits this sodium proton exchanger. So not only are you stimulating secretion but you are also inhibiting absorption. You get a double whammy. b. You can treat this fairly effectively by drinking something with glucose in it and salts (sodium bicarb) and water. You can make these rehydration drinks at home.

you can have infectious and/or chemical esophagitis

-Infectious: Candida, Herpes simplex, Cytomegalovirus -Chemical Ingestion: acids, alkalis, hot liquids, smoke -Radiotherapy

how is acid controlled coming into the duodeum?

- a Cl-/HCO3- exchanger, and this does the same thing as you would see in the salivary gland; it is absorbing Cl- in exchange for HCO3-, so it's pushing HCO3- out as it sees Cl- in the primary fluid -However, there is not enough HCO3- that you can generate with this thing in order to get sufficient buffering of the acid that's coming into the duodenum; -It's got 2 things that help it: the most important one of these is the CFTR Cl- channel, which conducts HCO3- also e. So HCO3- can leave the duct cell through that channel so you always have a constant supply of HCO3- coming out, accounting for some of extra HCO3- needed f. These cells are also packed full of carbonic anhydrase to generate HCO3- g. But even with lots of carbonic anhydrase, and with CFTR Cl- channel, that's still not enough HCO3- so pancreatic duct cells have a Na/HCO3- cotransporter at the basolateral membrane that pulls HCO3- in from the blood so you have enough HCO3- to empty into the duodenum ot buffer the acid h. What the cells can make themselves is not sufficient- need HCO3- from the blood, so that's why the transporter is at the basolateral membrane i. We also have carbonic anhydrase to make HCO3- and H+ → while the HCO3- is leaving over the apical membrane, you still need to get rid of the H+ j. Because of making so much HCO3-, there are 2 ways to get rid of H+: a Na/HCO3- exchanger and a H+ ATPase which pushes H+ out k. We also have a K+ channel, a basolateral Na+/K+ ATPase - all of this is being regulated by secretin l. This is the big job of secretin - to give you HCO3- rich pancreatic juice m. We have a secretin receptor coupled to cAMP, which will switch on this channel, which may or may not increase trafficking of this transporter up to apical membrane n. Cl- channel will also let Cl- out and allow for turnover of exchanger o. Unlike salivary ducts, which are impermeable to water, pancreatic ducts are very permeable to water p. You're pushing a lot of HCO3- out, which is negatively charged - this will pull Na+ by paracellular pathways → so now you've got NaHCO3 in the lumen of the duct which will also draw water q. You produce a large volume of isotonic NaHCO3- and that's exactly what you need to buffer all the acidic material coming from the stomach r. The CCK goes up when you eat a meal, stimulating digestive enzymes and secretin, which gives the pancreatic juice you need to wash enzymes out into duodenum s. So with these 2 hormones, get high volume of NaHCO3 fluid rich in digestive enzymes - you get it right when you need it to digest food - it's a beautifully-regulated system

Acid comes from ___-. It responds to ___ stimulation, which is ACh

-Acid comes from parietal cells. It responds to vagal stimulation, which is ACh

what causes the secretion of pepsinogen?

-It's under the control of the vagus nerve -Things that stimulate acid tend to stimulate all these things together. -It's also interestingly regulated by secretin. If secretin goes up, you get an increase in pepsinogen secretion. -the vagal efferents are probably more important than the role of secretin.

how does peristalsis occur in the small intestine?

-When chyme enters duodenal bulb, receptive relaxation occurs via vagus -Segmental propulsion: formation of moving pockets of chyme by close constriction of circular muscle. Segmentation contracts in middle of bolus -Results in mixing & aboral movement of contents -Controlled by myenteric nerves (fine tuned by ANS & hormones) -Peristaltic propulsion: contraction takes place behind the bolus of chyme and relaxation ahead of it. -Accomplished by simultaneous stimulation of excitatory & inhibitory neurons -Peristaltic rushes (due to irritation - results in diarrhea) & reverse peristalsis (vomiting)

in the small intestine and colon Cl is secreted via:

-a chloride channel, CFTR (cystic fibrosis transmembrane conductcance regulator). So when this channel is open/activated you will get Cl leaving the cell and it will pull Na with it via the paracellular pathway, now we have salt in the apical membrane and so we get secretion.

how does ferric iron get into the cell?

-we can only absorb Fe2+, so we have to change it This iron has two ways to get into the cell. 1.-One is that it will bind with this little protein called transferrin. Transferrin gets taken up by specific receptor, sitting at the surface of enterocyte. This is another example of receptor mediated endocytosis. And this complex taken up into lysozymes, it finally gets degraded, the iron is liberated. then it is bound to mobile ferrin, it goes across the cell and gets spat out to the basolateral membrane. -There it binds a transferrin in the blood. 2.-DCT1, for divalent cation transporter 1. It will bind iron and take iron into the cell. But it cotransports protons with it. This is another transporter that relies on that acidic microclimate that sitting on top of the cell in order to transport nutrient that we need into the cell. but once that iron gets into the cell, its handled the same way as the irons that come from the transferrin. It binds to mobilferrein, get transported out to the basolateral membrane into the blood and now binds to plasma transferrin and that ends up in liver.

how do we absorb peptides? (not oligopeptides)

-you can absorb proteins and polypeptides in a couple of ways: -as single AA via a transporter there are LOTS of different AA transporters, so one will only take aspartic and glutamic acid some are very specific, others have broader specificity (ie hydrophobic AA) but there are many different types. -BUT almost ALL of them use Na+ gradient to bring AA into the cell. This is similar to the sugar transport uptake of glucose and galactose so this depends on the inwardly directed Na gradient set up by the Na pump for this mechanism to work. -But most of our AA is not absorbed like this. Mostly absorbed as small oligopeptides.

- There is not much that really regulates the synthesis and secretion of bile salts from the hepatocytes -You could argue that the supply of cholesterol would regulate the synthesis - if you don't have much cholesterol you will have a hard time making bile salts

.

what are the 3 ways that proteins are absorbed by enterocytes?

1. as aa= dependent on Na+ using classic seconday active transport 2. di and tripeptides= depending on H+ gradient using PepT1 3. phagocytosis= either random or by M cells

fat digestion involves 3 phases:

1. emulsification into miscelles 2.action of lipase 3. absorption of fatty acids and monoglycerides by enterocytes

the 3 ways to absorb Na?

1. solute cotransport 2. Na/H 3. ENaC

what are the 3 things that regulate acid secretion?

1.)-Gastrin binding to CCKb receptors located on the plasma membrane. Remember that gastrin also binds to the enterochromaffin-like cells and stimulate those cells to produce histamine. 2.) histamine- coming from ECL cells, remember histamine is a paracrine, so those ECL cells are found right next to the parietal cells in the gastric gland. f. The third thing is acetylcholine. That's binding to peripheral muscarinic receptors. g. Gastrin and ACh both switch on the Ca-signaling cascade, the phospholipase-c IP3 protein kinase C signaling cascade. h. Histamine will work through adenylate cyclase, to produce cycic AMP, to switch on the PKA cascade. i. These three things together work very well, they're efficient. They're synergistic, so you don't need so much of each paracrine hormone to get a pronounced effect. -They will work together, they let you conserve these hormones, paracrines and such, because those are very expensive molecules to make, it lets you use much less of those secretagouges. They will work to cause that cell remodeling, stick that H/K/ATPase on the apical surface of the parietal cell.

pancreatic fluid is approximately___ per day

1.2-1.5

most blood coming to the liver is from :

1.Celiac artery to Hepatic Artery =Oxygen rich 2.Portal Vein=Little oxygen,Nutrient Rich

how does the vomiting reflex work?

1.Nausea, increased salivation 2.Diaphragm, thoracic and abdominal muscles contract against a closed glottis (retching -proceeds vomiting) 3.LES relaxes 4.High positive abdominal pressure forces gastric contents into esophagus 5.Brain co-ordinates synchronous contraction of inspiratory and expiratory muscles, reversing pressure gradient 6.High positive intra-thoracic pressure and relaxation of upper esophageal sphincter drives expulsion of vomitus out through mouth

what are the 3 causes of jaundice?

1.Pre-hepatic: i.e. increased hemolysis; Sickle cell anemia, thalassemia and hemolytic disease of the newborn 2.Intra-hepatic i.e. decreased conjugation or decreased uptake of bilirubin; hepatitis, cirrhosis, liver cancer; neonatal jaundice 3.Post-hepatic i.e. obstruction of bile flow; cholelithiasis, structural disorders of the bile duct, obstruction due to tumors

efferent activity in he swallowing reflex causes:

1.epiglottis to move toward the glottis 2. relaxation of the upper esophageal sphincter 4. pharyngeal muscle contraction- propels bolus through upper esophageal sphincter int esophagus

humans produce about ___ liters of saliva per day

1/2

the vomiting center is located next to the ___ in the brain

4th ventricle. it is very leaky it give you the prodromes= nausea, sweating, paleness, hypersalivation

what is chronic hepatitis?

6 months of duration of acute and Hepatocellular injury:apoptosis, necrosis, regeneration Portal inflammation Confined to portal tract or expanding portal tracts Fibrosis: portal, bridging Ductular reaction

obesity

Accumulation of adipose tissue of sufficient magnitude to impair health Assessed by BMI 18.5 -25 kg/m2 normal 25-30 - overweight Above 30 - obese 66% of adults in US overweight or obese Type 2 diabetes Cardiovascular disease Hypertension Fatty liver disease/gallstones Degenerative joint disease Cancer

3 hormones work together in the pancreas to stimulate pancreatic secretion?

Ach, CCK, secretin

what can cause cirrhosis?

Alcoholic disease Viral hepatitis Fatty liver disease (alcoholic and non-alcoholic) Biliary tract disease Hemochromatosis Wilson's Disease a1-antitrypsin deficiency Cryptogenic

Self Induced Protein/Energy Malnutrition

Anorexia nervosa PEM + endocrine abnormalities Anenorrhea, hypothyroidism Electrolyte abnormalities (cardiac arrhythmia) Bulimia Electrolyte imbalances Pulmonary aspiration Esophageal rupture

what types of colonic polyps are there?

Any elevation of intestinal mucosa Sessile/flat Pedunculated (easier to remove) Most polyps we seen on a daily basis are one of two general categories: - Hyperplastic polyps (some people call them "zits of the colon" and they are entirely benign) -Tubular adenomas (malignant/pre-malignant) -Various other things (mostly variants of both of these) are seen less commonly

what are the begnin tumors of the liver?

Benign: 1.Hemangioma - most common benign tumor (don't want to biopsy this= can lead to a lot of bleeding) 2.Adenoma - oral contraceptive use, glycogen storage diseases 3.Bile duct hamartoma (Von Meyenberg Complex)

what are some of the stomach tumors?

Benign: Mesenchymal lesions Leiomyoma Lipoma GIST (malignant potential) Hyperplastic polyps=Usually in setting of chronic gastritis Adenomatous polyps Neoplasm found in older adults Pedunculated or flat, up to 4 cm Fundic gland polyps

what does the cholera toxin cause?

Cholera is a bacteria that secretes a toxin which can enter across the apical membrane of the cell and binds to adenylate cyclase. The way it works is that it switches on adenylate cyclase and you get increased cAMP. You totally ramp up the production of cAMP. When you do that, you get increased activation of protein kinase-A, increase phosphorylation of the chloride channel. You get lots of chloride being pushed out of this cell. You also get lots of sodium going via paracellular pathways. You have lots of salt on the apical surface and lots of water. This gives you the diarrhea. - You die of hypovolemia.

what are some clinical conditions that can cause carcinoma of the stomach?

Clinical Conditions: 1.Atrophic gastritis-Environmental metaplastic or autoimmune gastritis, Damage of the gastric mucosa can lead to B12 deficiency This is because intrinsic factor is made by the parietal cells in the stomach (IF is needed for B12 absorption in the small intestines) H. pylori infection-MOST COMMON CAUSE 2.Behavioral/Environmental 3.Dietary: cured, smoked, pickled foods 4.Smoking 5.Genetic 6. Slightly increased risk with blood group A clinical signs: Asymptomatic until late in course Weight loss, abdominal pain, anorexia, anemia, melena Metastasis to liver, Lymph Nodes, ovaries 5 year survival <20% Prognosis depends on depth of invasion and metastatic extent Early detection dramatically improves survival

what is Eosinophilic Esophagitis

Clinical: Normal pH, Anti-reflux drug failure, Episodes of food impaction Histologically: Prominent intraepithelial eosinophilia with microabscesses

pyridoxine B6 deficiency

Coenzyme for enzymes involved in transamination, carboxylation, and deamination in metabolism of lipids and amino acids Present in virtually all foods and deficiency is rare Some drugs (isoniazid, estrogens, penicillamine) cause secondary deficiency Deficiency results in dermatitis, cheilosis, glossitis, peripheral neuropathy

how do the large intestinal reflexes work? coloncolonic?

Colonocolonic - distension of one area of the colon leading to relaxation of others; mediated by enteric & sympathetic nervous systems

what s GERD? GERD is not a primary motility disorder

Due to reflux of gastric contents through the LES into the esophagus resulting in symptoms, injury to the esophageal tissue, or both Most with mild-moderate (non-erosive) GERD have normal or increased resting LES tone Transient LES relaxation (tLESR) Treatment: Inhibit acid secretion, lifestyle changes, surgery If left untreated risk developing Barrett's esophagus and ADENOCARCINOMA it is not a primary motility disorder

how to test for HCV RNA by PCR?

Enzyme Immunoassay (EIA) anti-HCV: Main screening test Detects circulating antibodies - 2 months after exposure Advantages are ease of use, low variability, ease of automation & relatively low expense Sensitivity is 97-100% Positive predictive value varies on prevalence 95% with risk factors & elevated ALT 50% without risk factors & normal ALT Important for measuring the level of circulating HCV RNA (viral load) Diagnosis within 1 to 2 weeks after exposure Evaluation of patients with false positive EIA Diagnosis of patients who fail to develop antibodies Detection limit > 12 IU/ml

what increases gastric emptying?

Increases Emptying: Large volume Liquid (isotonic saline fastest) Relaxation in duodenum Decreased distensibility of orad portion Increased diameter of pyloric sphincter Increased contractions Small particle size

which carcinoma of the stomach is easier to remove?

Exophytic - fungating- like a bulb or mushroom flat- hard to remove looks like another fold of the stomach Excavated - erosive, looks black, may mimic peptic ulcers Infiltrating - Linitis Plastica-leather bottle stomach (Scary for pathologists looking at stomach biopsies, because it can be extremely subtle)

what is the function of the large intestine?

Function: Absorption of 0.5-1.5 L intestinal secretions that arrive at terminal ileum; only 100 ml excreted

what is the name of the channel that transports all monosaccharides to the blood?

GLUT2

Gastrin release, which is under the control of ___.

GRP And Histamine, from those ECL cells.

Which of the following secretions is the most acidic? -bile -gastric -intestinal -pancreatic

Gastric The correct answer is C. Gastric secretions are the most acidic, with a pH of 1.0-3.5. Intestinal secretions are mainly mucus, so their pH ranges from 7.0-8.0. Bile has a pH of approx. 7.8. The pH of pancreatic secretions range from 8.0-8.3.

Laboratory Evaluation of Liver Disease:

Hepatocyte function: -Serum albumin -Prothrombin time -Serum ammonia Hepatocyte integrity: -AST (aspartate aminotransferase) -ALT (alanine aminotransferase)- better indicator -LDH (lactate dehydrogenase) Biliary excretory function: -Plasma membrane enzymes -Serum alkaline phosphatase -Urine bilirubin -Serum bilirubin-Total (unconjugated +conjugated) and Direct (conjugated only)

vitamin C deficiency

Hydroxylation of procollagen Antioxidant properties Dietary sources: Milk, fruits (citrus), vegetables Deficiency: Scurvy Children: Bone disease: Cartilaginous overgrowth due to inadequate osteoid formation with bowing of long bones, depression of sternum and outward curvature of the ends of the ribs Hemorrhage due to vascular fragility: Subperiosteal hematomas and bleeding into joints Gingival swelling, hemorrhage, and infection Impaired wound healing

what is esophagitis?

Injury and inflammation of mucosa because of reflux, infection, chemicals (e.g. lye, drugs) -Reflux (GERD) is the most common cause of esophagitis it decreases LES tone, Sliding hiatal hernia Injury results from gastric acid Clinical symptoms: Dysphagia, heartburn, regurgitation Outcomes: Ulceration, stenosis, Barrett's esophagus, neoplasia Pathologic features of reflux: Hyperemia and basal hyperplasia, Increased intraepithelial eosinophils and neutrophils, Elongated LP papillae, low pH

what happens to intestinal waves to cause reverse peristalsis?

Intestinal slow waves are suspended during vomiting and replaced by bursts of electrical activity that propagate orally (reverse peristalsis). This reversal of the normal direction of gut contents allows chyme to enter the stomach. It is coordinated from the brain and relies on extrinsic nerves, rather than the ENS.

what turns pepsinogen into pepsin?

It's secreted as a precursor; it's not activated until you get out into the lumen of the stomach. There it's cleaved by acid to its active form, which is pepsin. It will also cleave it's self; Pepsin can cleave pepsinogen into pepsin. It's a protease. It will start chewing up peptides and polypeptides that you've eaten.

describe saliva. its secreted from ___ glands. it is regulated by___. its starts off as a ___ solution and is modified to end up as predominately ____. we produce about ___ l/day.

It's secreted from classical exocrine glands. -Regulation is exclusively neural -Composition is modified during passage down the duct. Starts off as isotonic NaCl, ends up as predominately NaHCO3- d. We produce about 1 - 1.5 L/day. That's important to remember. e. There are inorganic and organic components to saliva

what are the 3 most important driving factors in monosaccharide absorption?

Key elements are 1.SGLT1, 2.a negative transmembrane potential - that's the potential that's pulling the Na into the cell. 3.a low intracellular Na so the gradient is inwardly directed.

what are the malignant tumors of the liver?

Malignant 1.Metastatic - most common liver tumors 2.Hepatocellular carcinoma - most common primary malignancy (85% of all primary hepatic malignancies) Relatively uncommon in North America Men > women (b/c of alcohol) Peak occurrence between 50 - 70 years

what is marasmus?

Malnutrition caused by severe reduction in caloric intake Greater than 60% reduction in body weight adjusted for height and sex Loss of muscle mass and subcutaneous fat (somatic) Extremities appear emaciated Visceral compartment spared Associated anemia, multiple vitamin deficiencies, immune deficiency with increased infections

what is the function of chewing?

Mixes food with saliva Starts digestion by mixing with salivary amylase Reduces size for swallowing

HCV signs

Most persons infected with acute HCV are asymptomatic. If symptomatic with acute HCV, (constitutional, gastrointestinal & hyperbilrubinemia) higher chance of clearing virus. Symptoms when present are usually related to decompensated cirrhosis > 20 years with the virus.

Copper deficiency

Muscle weakness, neurologic defects, hypopigmentation, abnormal collagen crosslinking

Selenium deficiency

Myopathy, cardiomyopathy

describe phase 2 drug metabolism:

Phase 2: Conjugation a. The products of Phase 1 reactions are sometimes conjugated to a variety of different entities including: i. Glucuronic acid, acetyl, methyl, and sulfate; ii. Conjugation to glucuronyl is by far the most common. b. Example: i. Salicylic acid in phase II makes the drug water-soluble (due to the derivative). The more water soluble the derivative, the easier they'll be to excrete. c. Liver contains many conjugation enzymes: i. Glucuronyl Transferases→most common ii. Sulfate Transferases iii. Acetyl Transferases iv. Glutathione Transferases v. Methyl Transferases

prevention of HBV

Prevent transmission from infected persons through percutaneous or mucosal contact with infectious blood or body fluids (semen & saliva) Hepatitis B immune globulin (HBIg) Post-exposure prophylaxis Hepatitis B vaccine Pre-exposure prophylaxis Post-exposure prophylaxis All are produced with yeast & recombinant techniques to generate the HBsAg protein Two single-antigen vaccines are available (ENGERIX-B® & RECOMBIVAX HB®) & 3 combined HBV vaccines are available (COMVAX®: with Haemophilus influenzae type b (Hib); PEDIARIX®: with diphtheria, tetanus, acellular pertussis (DTaP) & inactivated poliovirus (IPV) vaccine; & TWINRIX®: with HAV) All are inactivated vaccines Adult Schedule is a 3 dose series: 0 (75%), 1(90%), & 6 months (96%) if you get a chronic infection: HBIg is used after liver transplant to prevent recurrent HBV Interferon Nucleoside analogues Lamivudine & Entecavir Nucleotide analogues Adefovir & Tenofovir These meds clear the infection, must take them every day to contine clearing. Meds very costly

how is primary esophageal peristalsis generated?

Primary esophageal peristalsis - generated by swallowing center in medulla; alternating contraction & relaxation propels bolus of food through esophagus to stomach

how do the large intestinal reflexes work? retropshinetric?

Rectopshinetric (defecation reflex) - feces entering the rectum will cause peristalsis and relax the internal anal sphincter; mediated by enteric nerves & parasympathetic nervous system

Swallowing Reflex is:

Reflex action; bolus of food triggers touch receptors in the pharynx. Pharyngeal receptors send afferents to swallowing center in medulla. Efferent activity causes: 1) epiglottis to move toward glottis (prevents food from entering trachea); 2) relaxation of upper esophageal sphincter; & 3) pharyngeal muscle contraction - propels bolus through upper esophageal sphincter into esophagus.

vitamin d deficiency

Rickets / Osteomalacia Vitamin D involved in bone mineralization Disorders due to a lack of vitamin D or a disturbance in its metabolism causing inadequate matrix mineralization Rickets: In children distinctive skeletal bone derangements are seen Craniotabes Frontal bossing Pigeon-breast deformity Lumbar lordosis Bowing of the legs Osteomalacia: In adults, bone formed during remodeling is undermineralized(Osteopenia Fractures)

how is secondary esophageal peristalsis generated?

Secondary esophageal peristalsis is initiated by mechanoreceptors sensing stretch of esophageal wall (in response to dry food bolus)

_____ stimulates bile flow and inhibit bile acid synthesis

The bile acids themselves

Which of the following is the site of the Na+-Bile acid cotransporter? A. Gastric antrum B. Gastric fundus C. Duodenum D. Ileum E. Colon

The correct answer is D. Bile salts are recirculated to the liver in the enterophepatic circulation via a Na+-bile acid cotransporter located in the ileum of the small intestine.

once proteins get to the small intestine they coming in contact with other proteases. what is the key enzyme here?

The key enzyme is TRYPSIN. Secreted as trypsinogen and it ends up In the SI, its cleaved by enterokinase. Enterokinase is responsible for converting trypsinogen to trypsin. Enterokinase is bound to the brush border of the SI. Trypsin is responsible for the activation all of these proteases you can see that the cleavage by enterokinase is essential to get good protein digestion. Trypsin is important for the cleavage of other enzymes.

chylomicrons are taken up in the liver how?

The liver via endocytosis takes up the chylomicron remnants from Space of Disse on the endothelium. Note: Actual chylomicron CANNOT be taken up in Space of Disse b/c they are TOO BIG.

how is the small intestine able to reabsorb so much salt and so much water?

The vast majority of water being reabsorbed over the course of the day is done as a consequence of solute cotransport. -in this case glucose, galactose, the uptake of nutrients, AAs, vitamin C, they are all Sodium coupled transport. - because we are moving sodium over this transporter, we are also going to move chloride and therefore we are going to end up with salt at the basolateral membrane. Water follows salt, so the more NaCl you can absorb the more water you can absorb. -because our small intestine is just filled with these nutrient cotransporters, we can absorb a lot of water as a consequence of daily operation of nutrient absorption. -So Na-dependent cotransport is probably the single most important way in which we are able to reabsorb lots of water. That combined with a relatively leaky tight junction that moves chloride

Megaloblastic Anemias

Two causes Folate deficiency Vitamin B12 deficiency Enlargement of erythroid precursors Hypersegmented neutrophils Bone marrow is hypercellular with megaloblasts Macrocytic RBCs

Undernutrition is the____ global cause of morbidity and premature death

Undernutrition is the single leading global cause of morbidity and premature death 1/3 of disease burden in developing countries is directly or indirectly due to poor general nutrition or deficiencies of specific nutrients leading to increased risk of infection In developed countries ischemic heart disease and cerebrovascular disease are the leading causes of death - associated with obesity.

what makes a vomiting patient hypokalemic?

You also lose K+. Remember you are talking about an H/K-ATPase and that K+ cycles across the apical membrane. If you are throwing up a lot you are going to lose some excess K that should be taken up. So you can develop hypokalemia. You could end up in cardiac arrest because in very low K you cannot fire AP well, so you can die from that. -Also, you have a loss of volume, an extreme volume contraction of extracellular fluid so you become hypovolemic

ammonia conversion occurs in

ZONE1 ammonia is a side product of eating proteins. anything that isnt absorbed in the small intestine, will get down in the colon and will be used to make urea.

what is sjorgens?

a progressive, autoimmune attack on your salivary glands more common in middle aged women. lymphocytes infiltrate salivary and lacrimal glands. associated with dry eye sicca syndrome. treatment- candies or sialouges, corticosteroids

how does hepatoxicity occur in tylenol overdose?

a. Generation of reactive metabolite by phase 1 - NAPQI b. Depletion of normal phase 2 reaction substrates/cofactors c. Oxidative Stress - GSH depletion d. Covalent binding to critical cellular proteins - loss of activity or function (mitochondrial proteins and ion channels) e. Mitochondria Dysfunction - ATP Depletion - Cellular Necrosis

what is cellbiose or cellulose made of ?

beta 1,6 linkages of 2 glucose

hepatitis D

blood tranmission (IV drug users) YES chronicity prevention: pre/post exposure immunization, reisk behavior mod. Therefore presents as either co-infection or superinfection to hepatitis B infection (never presents alone) -cannot replicate without HBV -Encodes one protein HDAg -HDV requires HBV to replicate HDAg -Co-infection - acquisition of both HBV & HDV at the same time (needle sharing) -Super-infection - acquisition of HDV infection after established HBV -Most US cases of HDV are found in IV drug users -HBV vaccination will protect against HDV

norepinephrine and epinephrine trigger release of ____

cyclic AMP and then protein kinase A signaling cascade and then neurotransmitters from the parasympathetic system, such as ACh, Norepi, and Substance P coupled to the calcium signaling cascade. I think it's enough to remember there are some that are coupled to calcium, and some coupled to protein kinase A. k. Both will converge are the apical membrane to increase secretion of those secretory granules, and to increase fluid production by opening up those chloride channels.

What is the best at stimulating the release of CCK?

fatty acids pH< 4.5

hepatitis E

fecal-oral transmission NO chronicity prevention: clean water clinical features similar with HAV

what is the main product of the pancreatic acniar cells?

inactive proteases: trypsinogen, chymotrypsin, proelastase, proprotease E, procarboxypeptidase A, procarboxypeptidease B, procolipase

Generally speaking, you have more ____ in the gallbladder than the liver

ion concentration

what is cirrhosis?

irreversible end stage of persistent liver damage characterized by bridging fibrous septa separating regenerative nodules of varying size occupying the entirety of the hepatic parenchyma. portal hypertension always occurs with cirrhosis

why is intrinsic factor important?

it helps you absorb vitamin b12

the main secretory product from the salivary acinar cells is___

na cl

Qualitatively, the _____is more important in the salivary glands.

parasympathetic If I cut the parasympathetic innervation, that gland is going to atrophy. The parasympathetic innervation is thought to be more important.

the __ cells of the stomach secrete acid

parietal, located in the gastric pits

bile acids are reabsorbed by the terminal ilieum via: ____

passive and secondary active transport

Cystic fibrosis patients can have:

people with cystic fibrosis often have something called pancreatic insufficiency, because the CFTR Cl channel doesn't work and so they can't wash out the digestive enzymes into the gut and have to take enzyme supplements to help them with digestion. These patients are often described as "failure to thrive" - they are underweight and tend to have stunted growth -They also have problems with bile because the CFTR Cl channel is also present in the bile duct cells -So patients with cystic fibrosis will have trouble with both pancreatic secretion and with bile secretion because of lack of function of the CFTR Cl channel

what are the endopeptidases?

pepsin, trypsin, chymotrypsin, elastase have specific peptide groups that they clip

-The regulation comes at the level of the bile duct, which works the same way as the pancreatic duct.It is primarily under the control of____

secretin is responsible for the fluid component of the bile, not the salts themselves -This bile is then stored in the gallbladder

what is sjrogen's syndrome (salivary ) is part of ___ syndrome that affects the lacrimal glands?

sicca

vitamin C is absorbed by:

small, water soluble, you need to have dedicated transporter, or a sodium dependent cotransporter

the lower esophageal sphincter is made of ___ muscle

smooth

Acid will stimulate the release of ____, which in turn will decrease the release of gastrin.

somatostatin

the upper esophogeal sphincter is made of ___ muscle

striated skeletal muscle

what does the rate of HCO3- secretion depend on?

the amount of Cl- HCO3- is secreted against its electrochemical gradient

an epithelium that is very leaky, has ____ electrical resistance from one side of the epithelium to the other

very little so low resistance means that it is very easy to move salt and water through

what is VLDL?

very low density protein, -VLDL are richer in triglycerides and has less cholesterol esters than are chylomicron remnants. d. Anything that causes an influx of FFAs to the liver (e.g., increased caloric intake, ethanol, estrogens), increase release of VLDL.

what allows bile being stored in the gallbladder into the duodenum via the common bile duct?

the sphincter of Oddi

if you have high numbers of tubular villous adenoma__

they can turn into colon cancer

when is gastric lipase inactivated?

when the fat reaches the duodenum. the pH gets much higher, but there is now pancreatic lipase present and bile salts to bind to.

what is the respiratory response for vomiting?

you are losing acid so there can be metabolic alkalosis, so there is decreased ventilation to try and raise the Co2 levels

where does gluconeogenesis occur?

zone 1

A lot of phase 2 reactions occur throughout the hepatocytes. c. Zone 1: phase 2 most occur is ___, but at Zone 3, Phase 2 is ____.

zone 1=gluconidation zone3=sulfation

what is lactose made of

galactose and glucose

vagus (Ach) innervation of the gallbladder causes?

gallbladder contraction

what is the MAIN job of secretin?

give rich HCO3- pancreatic juice to duodenum

what is the covering of the liver called?

glisson's capsule

what is sucrose made of?

glucose and fructose

in zone 3 most ammonia that is left there will be used to make:

glutamine

what is secretory diarrhea?

- an increased secretion. -VIP - it's a very good secreto-gouge. There are some people who have VIP-oma. There is a tumor that secretes VIP and they get water diarrhea. -There are certain irritant laxatives that irritate the mucosa and cause more secretion. A classic one is cholera. -You also have inflammatory diarrhea. Its really a consequence of the breakdown of the epithelial barrier. You see this with crohns disease and ulcerative colitis. You get inflamed, irritated mucosa and increased fluid production.

why doesn't the stomach digest itself?

- little layer of surface cells. They're producing bicarbonate and mucus and so what happens is that the mucus forms a meshwork and traps bicarbonate up next to the surface. -So any protons that are out in the gastric lumen that have the temerity to try to diffuse back to the surface epithelium to attack it are going to be buffered by that bicarbonate. -There are two sort of cells, there are the surface cells and we also have mucus cells called mucus neck cells, which are a bit lower. They also produce a thick and sticky mucus. -It's because you have this network of mucus filaments sitting on top of the surface epithelial cells that traps bicarbonate. So you have a bicarbonate buffering layer right next to the epithelium and that's what protects your epithelium from being damaged by the acid.

how does somatostatin work to turn off acid secretion?

- the gastric antrum, G cell that's secreting gastrin. -We're eating our meal and we're producing lots of gastrin and there are those small peptides and amino acids that are simulating gastrin -We're getting nervous input, so we're getting release of Gastrin Releasing Peptide, everything is just going along fine. -At the same time, we are inhibiting the production of somatostatin. -So somatostatin is low and gastrin is high, that's what you want. -You begin to empty out your stomach, the pH in the atrium begins to drop, because you're getting rid of those buffers, so the pH falls. -Now the somatostatin secreting cell suddenly sees a decrease pH. -This goes right next door to the G cell, and it turns that G cell off, so you stop producing gastrin. If you're not producing gastrin, your rate of acid secretion is not going to be high. -As you start secreting, as the pH continues to fall, you produce more somatostatin, less and less gastrin, by the time the stomach is emptied, you're not producing more acid. -So this is a very nice negative feedback loop between how these are regulated. -Somatostatin, when the pH falls, when the stomach is emptying, somatostatin secretion increases and turns off the production of gastrin. This is somatostatin acting as a paracrine.

pancreatic duct cells AND the fluid component of the bile use bicarbonate how?

- we have a Cl/bicarb exchanger, as well as the specialized Na/bicarb cotransporter bringing in bicarb from the blood and the CFTR Cl channel -The fluid component of bile is not coming from the hepatocyte, it is coming from the duct cells - very analogous to the pancreas - we have bicarb being pushed out into the lumen - this is pulling sodium and pulling water -It is being regulated principally by secreting, some by glucagon, and some by VIP

toxic drug induced hepatitis

-10% of cases of acute hepatitis 10-20% of fulminant hepatitis (compared to 1% for viral hepatitis) Most commonly due to antibiotics (flucloxacillin, erythromycin, nitrofurantoin, isoniazid etc.), psychotropic agents, anticonvulsants, anesthetics, oral contraceptives, acetaminophen Total number of know hepatotoxic drugs at least 1100 Pathologic lesions vary widely mimicking all forms of acute and chronic hepatitis Can cause neoplasm such as hepatocellular adenoma or angiosarcoma Ability to metabolize drugs has genetic component (i.e. cytochrome P-450)

GASTRIC MOTILITY: Receptive relaxation in orad region (this is a vagovagal reflex). Contractions are triggered by slow waves. In presence of food, frequency and strength of contractions and gastric emptying are influenced by mechanical, chemical, neuronal and hormonal factors.

.

When do we biopsy?

-Abnormal liver function test -Fever of unknown origin -Evaluation of jaundice of unclear etiology -Evaluation of portal hypertension and ascites -Evaluation of hereditary or metabolic diseases -Evaluation of abnormal serum studies -Grading and staging of chronic hepatitis -Monitoring the effects of new or establish therapy -Grading and staging of chronic biliary diseases -Confirmation of fatty liver and grading and staging of steatohepatitis -Diagnosis of space occupying lesion -Evaluation of transplanted livers

difference between cholecystitis and chronic

-Acute cholecystitis - a diffuse inflammation of the gallbladder, usually secondary to gallbladder obstruction -Chronic cholecystitis - inflammation of the gallbladder almost always associated with gallstones -Cholelithiasis - the presence of stones within the lumen of the gallbladder or biliary tree Affect 20 million Americans 5 F's - fair, fertile, female, forty, fat Clinical RUQ pain, vomiting, fever, jaundice Types of stones=Cholesterol - 85% and Black & brown bile pigment Calcium carbonate (rare)

sequelae of crohn's is

-Adhesion and fistula formation- can lead to intestinal obstructions -Peritonitis with or without abscess formation -Intestinal obstruction -Increased risk for carcinoma compared to general population -Segmental involvement with preference for terminal ileum: may involve any part of GI tract, including oral cavity (e.g. aphthous ulcers) -Lesions are segmental and TRANSMURAL -"skip" lesions -granulomas in half of cases -fistula formation -mucosal fissures, linear ulcers, cobblestoning -mucosal distortion and atrophy and mural fibrosis -Inflammation is acute and chronic dyplastic features may be present

Secondary Protein/Energy Malnutrition

-Advanced cancer patients -AIDS -Cachexia 50% of cancer patients: most prominent in patients with GI, pancreatic and lung cancers 30% of cancer deaths Weight loss, fatigue, muscle atrophy Atrophy of diaphragm

Gastric Motility is mediated by ____?

-As bolus enters stomach, receptive relaxation allows accommodation of meal - mediated via VIP. -Filling of stomach stimulates contractions, which mix food with gastric secretions to form chyme. -Waves of contractions that begin mid stomach move chyme toward antrum and pylorus. Because tone of pylorus is high, chyme moving from antrum undergoes retroplusion away from pyloric sphincter. -As contraction progresses to the antrum, relaxation of sphincter occurs & some chyme is ejected into duodenal bulb. As contraction reaches the pylorus, the sphincter is tightly closed and no further ejection of chyme occurs. -Release of CCK, GIP, and secretin decrease gastric emptying by constriction of the sphincter.

describe the ion secretion from pancreatic acinar cells:

-At rest, the secretion is predominantly NaCl, and this reflects the small amount of fluid that's coming from the acinar cells -You need some fluid from the acinar cells to wash those digestive enzymes out, to get them into the lumen of the duct - very similar to the salivary acinar cells -As secretion progresses, you find that you change the concentration of Cl- reciprocally with the concentration of HCO3- -pancreatic juice emptied out into the duodenum is predominantly NaHCO3; that is coming almost exclusively from the pancreatic ducts -Pancreatic acinar secretion is essentially identical to salivary gland acinar cell secretion - the basolateral membrane, you have the Na+ pump, a K+ channel, and the Na/K/Cl cotransporter working together like a Cl- pump to load cells with Cl- -When cells stimulated by CCK or Ach, get opening of Cl- channel on apical membrane, Cl- goes down it's electrochemical gradient; it takes Na+ with it, which takes water, so you have isotonic NaCl -that accounts for that first part of the secretion at rest, the isotonic NaCl

how does a parietal cell work at rest and active times?

-At rest, you would see that it's full of little tubular vesicles and lots of mitochondria. -When this cell is stimulated to secrete, little tubular vesicles fuse together and make canaliculi - these are deep invaginations into the parietal cell. The production of acid is due to the turnover of a protein called the H/K/ATPase. -These cells are contained in the glands, which are the deep invaginations into the mucosa, and the cells themselves have these invaginations of luminal membrane into the cell. -Takes about 10 minutes after seeing a stimulus to go from one stage to the next. There is a lot of cellular and cytoskeletal remodeling that has to be done. -This is a highly energy demanding process, so in order for this to work you have to have lots and lots of mitochondria to produce your ATP.

what is celiac disease?

-Autoimmune reaction to gluten protein in wheat -Causes diarrhea secondary to malabsorption -Impaired nutrient absorption secondary to loss of surface area of villi and intraep lymphs -Typically get greasy stools (fat malabsorption) -Can have an associated anemia (b12 not absorbed)

what is Wilson's disease?

-Autosomal recessive (mutation in ATP7B) leads to failure to excrete copper into bile, the primary route for copper elimination from the body -Mean age of diagnosis: 11-16 years old -Elevated serum ceruloplasmin -Pathology overlaps other acute and chronic hepatitis -Treat with copper chelation therapy -Toxic accumulation of copper in liver, brain, and eye -Kayser-Fleischer Ring in cornea -Toxic injury to basal ganglia, can lead to psychosis and Parkinson's like syndrome -Hepatic cirrhosis

what types of esophageal tumors are there?

-Benign tumors and polyps: Leiomyomas and fibrovascular polyps -Malignant tumors: Squamous cell carcinoma and Adenocarcinoma 1-2% of all US cancer deaths Gastrointestinal Stromal Tumors (GIST)= can be malignant

why do we have a gallbladder?

-Bile acids are nasty things - they are acidic, and are powerful detergents so that they can digest fat by creating micelles -You want some place where you can store them. They are expensive to make so you don't want to waste them all the time -You want to be able to store and concentrate them, so we stick them in a bag -So we have this small bag that contains highly concentrated bile - the gallbladder -There is a mucous layer that protects the epithelial cells of the gallbladder -It is regulated, so you only release the bile when you need to

Large Intestinal Motility: Modes: Haustrations/Segmentation Mass movement Defecation Functions: Mixing Propulsive motion Excretion

.

what happens in the Gi tract of cystic fibrosis patients?

-CFTR is the protein that is mutated in patients with cystic fibrosis. In cystic fibrosis you don't do this so you have very compromised intestinal secretions. Actually, in the mouse model of CF if the mouse is not treated it will die due to intestinal obstruction because there is not enough secretion to bathe the lumen appropriately, there is not enough fluid for the gut to work properly.

how does alcohol affect the hepatocytes?

-Complex and incompletely understood alters multiple metabolic pathways-oxidative stress and free radical production plays a major role direct and indirect toxicity

what is autoimmune hepatitis?

-Due to defect in suppressor T-cells which leads to disordered immunoregulation and production of autoantibodies Female predominance (78%) 1.9 cases per 100,000 individuals Serum aminotransferases more elevated than alkaline phoshpatase High titers of autoantibodies in 80% of cases such as anti-liver/kidney/microsomal antibody, anti ANA, Anti ASMA Associated with other autoimmune diseases (up to 34% of women, 17% of men) Hypergammaglobulinemia Absence of virus, drug, and alcohol related disease Indolent or severe precipitous course (fulminant) Histologic features overlap with chronic viral hepatitis Responds dramatically to immunosuppressive therapy lots of plasma cells are present int he liver

what is Hirschsprung's Disease?

-Enteric nervous system of the distal colon/rectum fails to develop during embryogenesis -Affects 1/5000 live births, due to mutations in chromosome 10 -Characterized by inability to have a bowel movement (pass stool) corrected by surgical removal of affected region

what happens in appendicitis?

-Epigastric, periumbilical, RLQ cramping pain and tenderness -Nausea, vomiting -Low grade fever -Leukocytosis can be caused by: -obstruction of orifice with secondary luminal -distention and bacterial invasion -Fecaliths (concretion of stool), tumor, parasites, foreign bodies, viral (e.g. adenovirus) -In many cases no obstruction can be documented Gross and histologic features: Fibrinopurulent exudate, microabscesses, transmural inflammation, possible perforation

liver function can be divided into Exocrine =__ Endocrine =___ Metabolic =____ Storage=___ Protection=____

-Exocrine e.g. Bile secretion -Endocrine e.g. clotting factor secretion -Metabolic e.g. fat metabolism -Storage e.g. glycogen, iron, vitamins -Protection e.g. detoxification, -Mcells (Kupffer Cells) these are specialized macrophages. They sit in the liver waiting for bacteria that have escaped the confine of the gut and making their way through systemic circulation b/c the portal vein goes straight through the liver. -This enables Kuffer cells to "catch" those bacteria before they get into systemic circulation.

how do we absorb the monosaccharide fructose once we have digested it?

-Fructose is a little different. It DOES NOT RELY on the Na gradient, but has its its own specialized transporter here called GLUT5. -GLUT5 only binds fructose and transports into the cell irrespective of the Na gradient (an example of facilitated diffusion) -But fructose will also exit the cell using GLUT2.

what is the main function of the small intestine?

-Function: Mixing, exposure of contents to absorptive surface, net propulsion

what are risk factors for colon cancer?

-High fat, low fiber diet (common American diet) -Genetics -Familial adenomatous polyposis -HNPCC-Hereditary non polyposis colon cancer Associated conditions: Inflammatory bowel disease Polyps precede most tumors Adenoma-carcinoma sequence

what affects secretion?

-Hormones and neurotransmitters (e.g. VIP, ACh, bradykinin, guanylin, serotonin) -Products of the immune system (e.g., histamine, prostaglandins, cytokines) -Bacterial toxins (e.g. Cholera, Yersinia) -Laxatives (e.g., bile acids)

what causes hypochloremia in vomiting?

-If you are throwing up HCl, not only are you losing H+ but you are losing Cl, so blood Cl drops.

what regulated the amount of chyme entering the colon?

-Ileocaecal sphincter regulates amount of chyme entering colon. Tonically closed but movement of chyme in terminal ileum relaxes the sphincter, allowing small amounts of chyme to enter colon -Ileocaecal sphincter is controlled by ENS, CCK, gastrin, and ANS Contraction of tenia coli cause the formation of sacs, called haustra. This is segmental propulsion. Haustra stay formed for long periods which stores the chyme as it is made into feces

what is barrett's esophagus?

-Intestinal metaplasia above esophagogastric junction -Velvety-red/salmon colored mucosa located above the junction -Present in 5-15% of symptomatic patients with chronic reflux esophagitis -More common in white males -30-100 fold risk of adenocarcinoma!!!!!!!!! With a dx of Barrett's - report must provide info on presence or absence of dysplasia (because it often precedes adenocarcinoma)

Only way to excrete cholesterol is via bile; if bile flow obstructed (late cirrhosis, Alagile syndrome, biliary atresia) leads to xs chol and xanthomas

.

Primary and secondary peristaltic contractions Relaxation of LES signaled by VIP and NO

.

We have talked a little about the paracellular movement of salts and water (moving through tight junctions) and I've mentioned epithelia being "leaky". So, when we talk about something being leaky or tight we are actually referring to the tightness of the tight junction. -So, the gallbladder is a very leaky epithelium. Although it has tight junctions between the epithelial cells, those tight junctions are not that tight. It is not as though those tight junctions are letting everything through, they are permeable to water and small electrolytes (small molecules).

.

how is b12 (aka cobalamin) absorbed?

-Intrinsic factor is a small mucoprotein, it's essential for the absorption of B12. -When we eat a meal, the first thing that happens is that the B12, which is also called cobalamin, it binds to proteins which are made in saliva, called R-proteins. -They bind to the R-proteins, and as you swallow your food and get to the stomach and the cobalamin is also being liberated by the acid in the stomach. -At the same time, you're secreting IF. -R-proteins bound to cobalamin bound to IF go down into the small intestine. -Once in the Small intestine, the R-proteins get digested by the digestive enzymes. This liberates the cobalamin. -The cobalamin now is looking for somebody to bind to, and it finds the IF. -This complex of Intrinsic Factor and Cobalamin go all the way through the small intestine until you get to the terminal ileum. At the terminal ileum, the whole complex is endocytosed. -It binds to a specific IF receptor, and the whole thing gets taken into the enterocyte. So it's receptor mediated endocytosis. -That's why you need the IF, without IF, no B12 absorbtion. It used to be that people who didn't have much Intrinsic Factor, would have to have shots of B12 in order to survive. Now there are intranasal preparations available. You can just inhale it, like you would with an asthma inhaler. -So the things that stimulate the production of acid also cause the secretion of Intrinsic Factor. So you'll get IF production when you're eating.

what are the main functions of the liver?

-It's crucial for regulation of metabolic homeostasis and energy metabolism -Takes up, processes, and stores ingested nutrients because it sees all the nutrients first.This is the FIRST organ in the body that sees what's coming out of the gut. -Synthesizes proteins, enzymes, and co-factors -Controlled distribution of substances to extra-hepatic tissues -Detoxifies and eliminates endogenous and exogenous compounds -Located between digestive tract/spleen and systemic circulation - receives all blood from digestive organs -Enterohepatic circulation - removal of hydrophobic substances via bile -Important immunologic function via removal of micro-organisms -Also has storage for vitamins, endocrine, exocrine function -Liver responsible for metabolizing and secreting most hydrophobic waste and toxic substances from the body via formation of bile and participation in the ENHC. -Immune function - vast blood flow through liver and large surface area of the hepatic sinusoids enable the liver to be a major organ for the phagocytosis and removal of foreign material -2% body weight in men and 3% body weight in women

other stomach malignancies

-Lymphoma (mostly MALT but can be Diffuse large b cell lymphomas, associated with H.pylori)a lot of them will respond to antibiotics -Neuroendocrine tumors -Gastrointestinal stromal tumors

what is Kwashiorkor

-Malnutrition with protein deprivation relatively greater than the reduction in calories -Common in malnourished children in Africa and Southeast Asia who often have almost exclusively carbohydrate diet -Also seen in disorders with protein loss (nephrosis) and following severe burns Loss of visceral protein Hypoalbuminemia Generalized edema Weight typically 60-80% of normal, but loss masked by edema Relative sparing of subcutaneous fat and muscle mass Skin: Areas of hyper and hypopigmentation with desquamation Hair: Loss of pigment Hair loss Enlarged fatty liver Defects in immunity

mass movements large intestine are very slow

-Mass movements- very slow segmental propulsion - 1-3 times/day -Peristaltic in nature, but contractions occur over a longer period in time, force the feces to travel through the descending colon to rectum -Stimulated by parasympathetic nerves (vagus & pelvic nerves), gastrin & CCK

what affects absorption?

-Mineralocorticoids (e.g., aldosterone) -Glucocorticoids (e.g., dexamethasone) -Somatostatin (stimulates NaCl absorption) -Opiates (e.g., enkephalins, morphine)- shut down secretion? -Neurotransmitters (e.g., norepinephrine, enkephalins)

what is psuedomembranous colitis?

-Overgrowth of toxin-producing Clostridium difficile Prior broad-spectrum antibiotic Rx Grayish superficial necrotic exudate Fever & bloody diarrhea

The acinar cells make enzymes, the duct cells make fluid!! so that's a difference between pancreas & salivary gland

-Roughly 60% of the bicarbonate that you need is coming from the pancreas, all from the duct cells; very little fluid is coming from the acinar cells -About 40% of this bicarbonate that you need comes from the surface villi cells in the duodenum itself

what is Nonalcoholic Steatohepatitis

-Similar pathologic findings to alcoholic liver disease, but without history of alcohol intake -Up to 5% of the US population may be affected (getting more prominent in US) -Patients may be asymptomatic or show mild nonspecific fatigue and right upper quadrant pain -Associated with metabolic syndrome (obesity, arterial hypertension, dyslipidemia, microalbuminemia) and diabetes -Pathogenesis is complex; insulin resistance and oxidative stress are important factors -Can progress to fibrosis and cirrhosis

what is the composition of gastric juice?

-Similarly to salivary glands, it's a two component secretion -Instead of modifying the secretion, these two components come from different regions of the gland. -At rest, what you're going to get predominantly is Na and Cl.This secretion comes from surface epithelial cells. This is just your baseline, everyday, house keeping type gastric secretion. -It's going to be isotonic NaCl. -When this gland is stimulated, those parietal cells remodel, they start producing HCl, and what happens is that the proportion of H and the proportion of Chloride now increase and so at maximum secretory rate, you're producing essentially 100% HCl - the magnitude of secretion from parietal cells is what gives you the hydrochloric acid. It's not that the surface epithelial cells are doing anything different, they're just being overwhelmed by the amount of acid being produced. - the Na in that secretion drops. It's not that the epithelial cells stop secreting, it's just that there's so much more acid present for the given volume that it looks as though the Na concentration drops.

what is ulcerative colitis?

-Slightly more common than CD, and affects men and women with equal frequency Presentation is similar to CD, but often more severe with more frequent bleeding and pain Extraintestinal manifestations: Polyarthritis, sacroiliitis, ankylosing spondylitis Uveitis Erythema nodosum Primary sclerosing cholangitis Diffuse and continuous involvement beginning at the rectum chronic inflammation, crypt abscesses Mucosal hyperemia to extensive ulceration pseudopolyp formation perforation is rare Chronic changes atrophy, submucosal fibrosis, mucosal distortion Increased risk of carcinoma compared to general population

what inhibits acid secretion?

-Somatostatin will inhibit acid secretion. In this context, where it's working at the parietal cell, somatostatin is working as a hormone. -It's most effective role in inhibiting acid secretion is going to be working as a paracrine, where preventing the signal release of gastrin.

each sugar has its own enzyme what are the names?

-Sucrase digests sucrose -Maltase digests maltose and maltotriose -Isomaltase digests a-dexrin -Trehalase digests trehalose -Lactase digests lactose The final products of those digestive processes are the basic monosaccarides: glucose, fructose, galactose

what happens in acute hepatitis?

-Swollen, congested liver -Apoptosis of hepatocytes -Acidophil bodies- main finding -Ballooning degeneration -Steatosis -Hepatocyte necrosis:focal, piecemeal, bridging -Inflammatory infiltrates -Portal and lobular based -Regenerating hepatocytes

what is gluconeogenesis?

-The Chief Non-Carbs precursors: -Lactate and pyruvate -TCA cycle intermediates -Carbon chains of some AA -All substances must be converted to oxaloacetate. -Extra: Occurs when dietary sources of glucose are low and when the liver is depleted of glycogen -Maintains blood glucose in fasting person to support metabolism in high glucose utilizing tissues - RBCs, kidney, lens, cornea, brain, etc... -Liver takes up glucose after meal and converts it to glycogen; approx. 80g glycogen is stored. -Glycogen synthesis is promoted by insulin and inhibits gluconeogenesis. Summary: During a well-fed state: you don't have to make glucose form non-carbs intermediates. So, gluconeogenesis is inhibited. -During fasting, glucagon (true hormone secreted by pancreas) activates breakdown of glycogen to free glucose; a key enzyme is glucose -6-phosphatase, which is not found in brain or muscle. glucose -6-phosphatase are found ONLY in LIVER. -Glucagon also stimulates gluconeogenesis and ketogenesis. d. In a "fight or flight" situation, epinephrine activates breakdown of glycogen in liver (and muscle).

what is the alakaline tide?

-There is actually so much bicarbonate that's being produced in a secreting stomach that the blood coming from the stomach when you're eating will actually be slightly alkaline. You might see that referred to as the alkaline tide. Once that blood gets into the systemic circulation, of course that pH is buffered away. -Initially, as it comes from the stomach, the pH will be alkaline, and that purely reflects the amount of protons that are being secreted over the apical membrane. n. You have to secrete lots of protons to generate a pH of about 1

how doe parietal cells secrete acid?

-This is our parietal cell. This is the mechanism of acid production. - the business end of this process is the H/K/ATPase. -this exchanges 1 K for 1 H. So it's a 1:1 exchanger. -Because it's producing protons out into the lumen of the stomach, where there are lots of protons when you're secreting, it's secreting protons against a gradient. This is active transport, it's the only way you can get this to work. -Also out at the apical membrane you have a K-channel. This K-channel lets K cycle futilely across the apical membrane. So K leaves, comes back in over the ATPase, and that enables this exchanger to turn over. -you need to have the H/K/ATPase and a K-channel at the surface. -The other thing you need, if you're going to make HCl, is a way for the Cl to get out. So also at the apical membrane you're going to find a Cl-channel.

what is the small intestine transit time?

-Transit Time: Approximately 2 h to empty, with contents moving at 5 cm/min

risk factors for getting HAV?

-Travelers to regions with intermediate or high rates of HAV -Sex contacts of infected persons -Household members or caregivers of infected persons -Injection & non-injection illegal drug users -Persons working with primates susceptible to HAV infection

describe the slow waves:

-Undulations in resting membrane potential of the smooth muscle cells -Extend from stomach to distal colon -Generated by the interstitial cells of Cajal (situated throughout the GI tract between the longitudinal & circular muscle layers) -Under resting conditions, slow waves undulate between -70 & -80 mV & do not cause contractions -If slow waves are depolarized, by nerve activity or hormones, & the peak of the slow wave exceeds the threshold potential (-40 mV), cells will generate an action potential. The greater the depolarization, the more action potentials & increased strength of contraction -Inhibitory neurons release VIP and NO, hyperpolarizing the slow waves & relaxing smooth muscle

why is vitamin c important?

-We need vitamin C to be able to absorb iron, because we absorb ion as ferris iron (Fe2+) -Vitamin C, one of the things vitamin C will do is to reduce ferric ion (Fe3+) ion to Ferris ion (2+).So we really need vitamin C to absorb iron We get irons from various sources. We eat it, some vegetables are very rich in iron as you probably already know, chocolate is quite rich in iron. -And we can also get it as heme iron, which are the break-down from blood cells, and we get heme iron from eating red meat.

when you start to eat, what controls the duodenal/ intestinal pancreatic secretion?

-What happens when you're starting to eat? The whole thing just kicks off -Acinar cells - we have CCK being released from I cells in duodenum, released in response to all foodstuffs, particularly to fats -CCK is a hormone - it goes into the blood, travels the systemic circulation, finds its receptor on the acinar cell, and ramps up the production of digestive enzymes -At the same time, we have secretin, which is very sensitive to acid, but is probably mostly stimulated for release by foodstuffs -So you're beginning to get food in the duodenum, which will cause secretin to be secreted, enter the bloodstream, and find its receptor on the pancreatic duct cell, so it ramps up the production of pancreatic fluid -Now you have lots of enzymes, lots of HCO3- - rich pancreatic fluid emptied into the duodenum and buffering the acidic chyme and promoting digestive enzymes to start food digestion

what is the job of colipase?

-colipase stabilizes the interaction of lipase with the fat droplets. -Once colipase is there, you can now bind your lipase and start digesting the fats start splitting up the triglycerides into free fatty acids and mono-glycerides. -This is the reason why lipase gets secreted from the pancreas without precursor because pancreatic cells, the membranes are bilayers, they are lipid bilayers just like any other cell, and lipase should digest the bilayer. The reason why it doesn't do a good job is because there is no colipase around it's colipase, if you remember from yesterday is secreted as a precursor. -and Procolipase is not cleaved until it get into the duodenum where it sees trypsin. So trypsin has to cleave procolipase to colipase so that colipase can stabilize this interaction with lipase on the surface of the fat droplet. -So now we have lipase digesting these fats making these free FA. -What is happening is that these guys are these free fatty acids digested and they attract more and more bile salts. -They thin out to make a very large vesicles and then they form micelles as they attract more and more bile salts to make, to bile salts to reach this critical micelle concentration when they can form micelles with the fat.

in order to secrete bicarbonate in the small intestine, we need to___

-couple the CFTR with Cl-/HCO3- exchanger - In the duodenum we are receiving HCO3 from the pancreas but the duodenum must also secrete its own HCO3 because it has to buffer the acid entering the intestine. -In the duodenum particularly you will see these mechanisms for secreting HCO3 and this is just the Cl/HCO3 exchanger that we have already discussed - The HCO3 is coming out and Cl is recycling through this mechanism. -We also need to be able to buffer that HCO3 so we have a little bit of H+ coming out here and you get some Na being reabsorbed by the turn over of this NHE (Na/H Exchanger) at this point so the Na will get removed by the pump. It is thought here, this is some HCO3 being brought into these cells by the NBC1. Remember we talked about Na/HCO3 cotransport in the pancreas, so in the duodenum you try to get even more HCO3 into these cells via operation of this transporter. Here we also have carbonic anhydrase generating HCO3 for the turnover of this transporter. -CFTR is also permeable to HCO3 as well as Cl so that is another way to get HCO3 out. These are all mechanisms to try to increase the secretion of HCO3 into the lumen.

what is bilirubin?

-formed from degradation of hemoglobin -binds to plasma albumin, actively extracted from blood by liver and conjugated to glucuronide -secreted into bile; deconjugated to urobilinogen; excreted and some recirculated to liver -urobilinogen oxidized to urobilin and stercobilin

How does the gallbladder concentrate bile?

-gallbladder cell is a typically example of a leaky epithelium - you don't have to move much salt in order to move appreciable amounts of water. This is good if your job is to concentrate something - at the apical membrane, we are moving Na in and Cl in -With salt comes water following paracellularly -We are moving Na and Cl through transcellular pathways which are exiting over the basolateral membrane -our Cl channel lets Cl out. Na is removed by Na/K ATPase. Water is following it -You can move lots and lots of water without moving much salt - this lets you concentrate half a liter of bile to store it in something with a capacity of 50 mL.

salt and water can be transported via the Na/H exchanger how does this work?

-if we are moving H+ we also have to move something to buffer those H+ so what we move is HCO3-. This is how we get absorption of NaCl in a situation in which we are using the NHE (Na/H Exchanger). -So now we move Na into the cell, H+ out of the cell, and in order to buffer those protons this exchanger is coupled with something that moves Cl and HCO3-. We are moving Cl- into the cell and HCO3- out of the cell. -This Na gets removed via the Na/K pump, the Cl might leave over a K+ cotransporter, it is not really clear yet how the Cl gets out of these cells. But this is another way in which we are absorbing net NaCl. -Again, if we are moving NaCl we are moving water. You see this in the small intestine and in the colon. Obviously the solute cotransporter is pretty much confined to the small intestine. -You don't need to worry about what these things are actually called here.

How might you treat cholera?

-rice water with salt thrown in -What is starch digested to? Glucose. How does glucose get into the cell? Glucose transporter then water follows. The starch is digested to glucose, glucose is co transported with sodium. Sodium is moved into the cell and chloride follows. Then water follows that. You can actually treat people very effectively just by feeding them pourage. SALT, WATER AND GLUCOSE. That is using the physiology of the system. You can change from net secretory to net absorptive. If you can maintain their fluid volume, they are fine. So this is how cholera would work with a massive activation of the chloride channel.

Every zone has glutathione

.

ii. Phase 1 rxns - drugs are modified and made more polar iii. Phase 2 rxns - partially polar metabolites generated via phase 1 reactions, react with the phase 2 enzymes which conjugate these substances to endogenous metabolites thus making them extremely polar so that can be efficiently excreted by the body. iv. Interesting that there is zonal distribution of these phase 1 and phase 2 enzymes. v. PV distribution of CYP450 - toxicity in zone 3 vi. Hepatocytes in zone 3 important for general detoxification and biotransformation of drugs. vii. The zonal distribution drug toxicity manifested as cell necrosis may be attributed to zone 3 localization of the enzymatic pathways involved in the biotransformation of toxicants by oxidation, reduction, and hydrolysis reactions.

.

the gallbladder does not secrete bile, it releases it. THE LIVER SECRETES BILE.

.

the more bile salts that you have, the more you can secrete them. So they are sometimes called choleretic - bile salts will increase their own secretion. -The more bile salts you recover, the less you have to make. So they inhibit their own synthesis

.

what linkages does amylase cleave?

1,4 linkages it will not cut 1,6 linkages or branch point linkages or terminal 1,4 linkages. you will never get free glucose from amylase

what is maltose made of? isomaltose?

2 glucose 2 glucose in 2 alpha linkages

You will lose about _ of bile salts per day, and probably have a pool of about _. So you synthesize about the same amount as what you lose a day to make up for what you lose

600 mg, 3 g

A 60-year old man presents with hematemesis, melena, and signs of circulatory collapse. He has a 20-year history of burning mid-epigastric pain and tenderness relieved by food, milk, or antacids. Also he has been taking high does of NSAIDS to relieve the pain from long standing arthritis. Esophagogastrodudodenoscopy reveals a peptic ulcer in the upper duodenum. Which of the following is an important association of duodenal peptic ulcer disease? A. Barrett esophagus and columnar intestinal metaplasia of esophageal squamous epithelium B. Evolution of carcinoma as a likely sequela C. H. pylori infection D. Hiatal hernia and incompetent lower esophageal sphincter E. Pernicious anemia and achlorhydria

A. Barrett esophagus and columnar intestinal metaplasia of esophageal squamous epithelium B. Evolution of carcinoma as a likely sequela C. H. pylori infection The correct answer is C. Dudodenal ulcers are associated with hyper secretion of acid and are closely related to gastric H. pylori infection. H. pylori increase gastric acid secretion and impairs mucosal defenses. D. Hiatal hernia and incompetent lower esophageal sphincter E. Pernicious anemia and achlorhydria

A 13-year old boy presents to the pediatrician with extrapyramidal signs, including a resting a kinetic tremor. An eye examination demonstrates the presence of Kayser-Fleicher rings, and his lab studies demonstrate elevated liver enzymes. It is likely that the patient's condition is associated with the accumulation of A. Copper B. Councilman bodies C. Eosinophiic hyaline inclusions D. Glycogen E. Iron

A. Copper The correct answer is A. Wilson diseases is a hereditary condition associated with the accumulation of copper in the liver, brain, and eye. Accumulation of copper in the Descement membrane of the eye results in the pathognomic lesion known as Kayser-Fleischer ring. Accumulation in the liver results in cirrhosis. Accumulation in the brain results in motor symptoms. Councilman bodies are apoptotic hepatocytes that were first identified in yellow fever. Eosinophilic hyaline inclusions and Mallory bodies are seen in alcoholic liver diseases. Glycogen accumulates in the liver in glycogen storage diseases. Iron accumulates in hemochromatosis. B. Councilman bodies C. Eosinophiic hyaline inclusions D. Glycogen E. Iron

presentation of carcinoma of the colon

Asymptomatic for long periods Weight loss, occult bleeding, anemia, changes in bowel function, fatigue Location determines likely presentation Left colon (descending) - early obstruction (changes in bowel habits, crampy pain) Right colon (ascending) - iron deficiency anemia clinical progression: 1. Normal mucosal surface and crypt epithelium 2. Aberrant crypt foci (histologic, no clinical correlate) 3. Adenomatous polyp (usually tubular adenoma) 4. Invasive carcinoma 5. Metastatic carcinoma Well defined clinical and correlating histologic progression.

what happens in a Alpha-1-antitrypsin Deficiency

Autosomal recessive Results in non-secretion of AAT (protease inhibitor) from hepatocytes Pulmonary emphysema Hepatic cirrhosis

Which of the following substances must be further digested before it can be absorbed by specific carriers in intestinal cells? A. Fructose B. Sucrose C. Alanine D. Dipeptides E. Tripeptides

B. Sucrose The correct answer is B. Only monosaccharides can be absorbed by intestinal epithelial cells. Disaccharides, such as sucrose, must be digested to monosaccharides before they can be absorbed. Proteins are hydrolyzed to amino acids, dipeptides, or tripeptides and all three forms are transported into intestinal cells for absorption.

hepatitis C

Blood transfusion YES chronicity prevention:blood donor screening and risk behavior modification 25% of HIV patient also have hepatitis C Portal lymphoid aggregate RNA Virus 75% of persons with HCV are unaware that they have the virus. Approximately 12,000 deaths each year in the US secondary to chronic HCV

what are chylomicrons?

Chylomicrons are protein bonded vesicles, they are too big to diffuse from the cell so they leave the cell via endocytosis and go into the lymph -that they get exocytosed from basolateral membrane, particular into the basolateral spaces here, and then they get taken up by the lymph vessels or the lacteals that run through the middle of the villi. -They are much too big to enter the capillary. They have to taken up by the lymph vessels, by these lacteals -Once they are in the lymph vessels, they get carried into the lymph system and get dumped into the thoracic duct -Chylomicrons are little protein-coated vesicles composed mostly of triglycerides with a little bit of cholesterol and phospholipids

vitamin K deficiency

Cofactor for microsomal carboxylase necessary to produce certain clotting factors and other diverse proteins Vitamin K is recycled (oxidized and then reduced to active form in liver) so that daily requirement is low Endogenous intestinal bacteria synthesize Widely available in Western diet Fat malabsorption Alteration of intestinal flora by antibiotics Neonatal period (reserves low, bacterial flora not well developed) Diffuse liver disease Bleeding diathesis: Hematomas, hematuria, melena, ecchymoses, bleeding from gums Neonatal bleeding may be severe with intracranial hemorrhage Routine vitamin K prophylaxis given to all newborns

niacin deficiency

Component of NAD and NADP, central role in cellular metabolism Dietary source: Grains, legumes Synthesized endogenously from tryptophan Deficiency can result from dietary insufficiency or from tryptophan deficiency In developed countries, deficiency seen in alcoholics and patients with chronic illness Pellagra Three Ds Dermatitis: bilaterally symmetric thickening an roughening of skin with scaling and infection Diarrhea: atrophy of GI epithelium with inflammation and ulceration Dementia: degeneration of neurons in brain and tracts in spinal cord

what is Crohn's disease?

Crohn's Disease and Ulcerative Colitis: Abnormal local immune response against the normal flora of the gut and self antigens in genetically susceptible individuals Common features: Inflammation, mucosal destruction, intermittent bloody diarrhea Classical differences between the 2 crohn's= throughout the gi tract -Higher prevalence in the West and among -Caucasians (1-3 per 100,000 incidence) and rising -Usually presents in second to third decades crohn's patients present with: Variable presentation Progressive, episodic, recurrent bloody diarrhea and abdominal pain Extraintestinal Manifestations iritis, uveitis, polyarthritis, sclerosing cholangitis, renal disorders

vitamin E deficiency

Dietary Source: Vegetable, grains, nuts, dairy products, fish, meat Dietary deficiency rare Absorption requires normal biliary and pancreatic function Vitamin E is stored throughout the body, mostly in fat, but also in liver and muscle Acts as an antioxidant to scavenge free radicals Almost impossible for dietary insufficiency to occur Fat malabsorption Abetalipoproteinemia: Autosomal recessive disorder with abnormal vitamin E transport Rare autosomal recessive syndrome of impaired vitamin E metabolism Nervous system: Axonal degeneration in posterior columns of spinal cord Depressed tendon reflexes, ataxia, dysarthria, loss of position sense, loss of pain sensation Disorders of eye movement Red blood cells with shorter half life - may lead to anemia

Vitamin A deficiency

Dietary Sources: Liver, fish, eggs, milk, butter, yellow and leafy green vegetables (carrots, squash, spinach) Retinol transported in chylomicrons to the liver for storage Greater than 90% of vitamin A reserves stored in the liver (normal storage enough for 6 months) Function: Maintaining normal vision in reduced light Potentiating the differentiation of specialized epithelial cells Enhancement of immunity Synthesis of rhodopsin from retinol Photon striking rhodopsin generates nerve impulse Differentiation of mucus-secreting epithelium Deficiency leads to squamous metaplasia and keratinization Plays a role in the acute phase immune response Reduced tissue availability Stimulates immune system, possibly through metabolite 14-hydroxyretinol Occurs with generalized malnutrition or with malabsorption of fats Rare in the US First sign is reduced vision particularly in low light (night blindness) Xeropthalmia (dry eyes) due to conjunctival alterations Bitot spots: buildup of keratin Erosion of the cornea (keratomalacia) Eventual blindness Squamous metaplasia of respiratory epithelium Increased pulmonary infections Squamous metaplasia of urinary tract Renal stones Skin Plugging of ducts leading to papular dermatosis Increased infections

what is hemaotchormatosis?

Excessive iron accumulation in tissues- direct hepatotoxin Liver - can lead to cirrhosis Pancreas - diabetes mellitus Skin - hyperpigmentation Elevated serum iron & reduced TIBC (total iron bound concentration ) Treat with phlebotomy to remove iron Types -Primary - multiple gene defects identified (autosomal recessive disorder due to mutations in HFE gene) -Secondary - transfusion or ineffective erythropoiesis

what is a fundic gland polyp?

FGPs occur both sporadically (more common) and in association with FAP - some think FGPs are related to use of PPIs (proton pump inhibitors)

what is familial adenomatous polyposis?

Familial Adenomatous Polyposis (FAP) Autosomal dominant mutation of APC gene 100% chance of malignant transformation Requires surgical intervention

what is the fecal composition of the large intestine?

Fecal composition: Bacteria, dead cells, indigestible material, water (75%), bile salts

what are the types of gastritis?

Gastritis: Can be active or inactive -Active=Helicobacter pylori is the most common cause of CHRONIC gastritis -Reactive gastropathy (aka - chemical gastritis)-common cause=alcohol, NSAIDs, and bile reflux Iron pill gastritis- Atrophic gastritis-Environmental metaplastic or autoimmune gastritis, Damage of the gastric mucosa can lead to B12 deficiency This is because intrinsic factor is made by the parietal cells in the stomach IF is needed for B12 absorption in the small intestine

how do the large intestine reflexes work? gastrocolic?

Gastrocolic - mass movements in colon after a meal; mediated by parasympathetic nervous system and CCK & gastrin

what are the gastroileal Small Intestinal Reflexes?

Gastroileal - Increased segmentation in the ileum in response to gastric emptying; mediated by the parasympathetic system and CCK & gastrin

What exactly is the defining difference between as primary and secondary bile acid?

If a bile acid still has the 7-OH group, it is primary. If the 7-OH has been stripped off, it is a secondary.

CAUSES OF MALNUTRITION IN US

Ignorance of nutritional requirements Poverty Chronic alcoholism/drug abuse Acute and chronic illness Self imposed dietary restrictions

riboflavin B2 deficiency

Important in a variety of oxidation-reduction reactions Incorporated into certain mitochondrial enzymes Dietary source: Meat, dairy, vegetables Deficiency in developed countries usually associated with alcoholism or debilitating disease May be seen in individuals who avoid dairy as this is a major dietary source Cheilosis/Cheilitis: Cracks and fissures at the angle of the mouth Glossitis with atrophy Corneal inflammation with ulceration Scaling dermatitis

how does the ENaC work and were is it located?

In the colon, it will pull Na into the cell which will then leave via the Na/K pump and Cl will follow via a paracellular route. We are ending up with NaCl at the basolateral membrane and water at the basolateral membrane. -Aldosterone works and helps you conserve salt by putting ENaC channels into the salivary ducts if you are dehydrated. The same thing happens in the colon, you will also get more ENaC expressed in the colon if I put you in the desert. -Mostly, you get ENaC expressed in the kidney and that is where it is going to do most of its salt conserving/salt absorbing work but you will also see increased expression in the colon and salivary ducts.

Protein-Energy Malnutrition

Inadequate dietary intake of protein and calories to meet the bodies needs Affects up to 25% of children in developing countries In severe forms, malnutrition is obvious Mild forms diagnosed using height/weight standard tables Child weighing less than 80% of normal is considered malnourished. Two major forms Marasmus Kwashiorkor

HBV risk factors

Infants born to infected mothers Sex partners of infected persons Sexually active persons not in a monogamous relationship (>1 partner/six months) Men who have sex with men Intravenous drug users Household contacts of persons with chronic HBV infection Health care & public safety workers at risk for occupational exposure to blood or blood-contaminated body fluids Hemodialysis patients Residents & staff of facilities for developmentally disabled persons Travelers to countries with intermediate or high prevalence of HBV infection

what are the intestinointestinal small intestine reflexes?

Intestinointestinal - if one area of the small intestine is over-distended (e.g, by bacterial infection), the rest of the small intestine will relax, causing cessation of motility; mediated by enteric & autonomic nerves

Iron Deficiency Anemia

Iron deficiency most common form of nutritional deficiency Serum ferritin is a good indicator of adequacy of iron stores Low dietary intake very rarely the cause in the US Malabsorption Pregnancy and infancy Chronic blood loss (colon CA) two types:Hypochromic Microcytic Most often asymptomatic Nonspecific manifestations Diagnostic criteria Low Hb Low MCV Low serum ferritin Increased total iron-binding capacity Response to iron therapy

The liver plays a central role in lipid transport and metabolism:

It facilitates digestion and absorption of lipids via the production of bile; It synthesizes FFAs, TGs and PPLs; It converts fatty acids to ketone bodies; It plays an integral role in synthesis and metabolism of plasma lipoproteins.

what is the main function of the lower esophageal sphincter?

Lower esophageal sphincter (LES) has high resting tone to prevent acid reflux. Maintained by enteric nerves, vagal cholinergic fibers, and hormones (gastrin) As bolus nears LES, non-cholinergic vagal fibers act on inhibitory interneurons to release VIP & NO, causing relaxation of LES. Bolus then enters stomach

how are the lipid absorbed in the enterocyte?

Mechanism of Lipid Absorption by the Enterocyte -the emulsified droplet is coated with layers to layers of bile salts. The lipase is working, so gradually this droplet gets smaller and smaller. Eventually is going to pinch off. -This is going to make an multi-aveller vesicles that will attract more and more bile acids. It will thin out and it become larger now. And will be called a unionmeller vesicles. As it attracts more and more bile acids, these then get converted into micelle. -They are coated with bile acids and sequestered inside the fatty acids, cholesterols and etc. -You need to get the miscells into the enterocytes somehow. -What happens is that as it being washed down the gut, occasionally it is going to run into the brush border of enterocytes. When it does that it is going to meet the acidic microclimate. (Remember about protein digestion, how you have an apical Na/H exchanger and that is important for acidifying the surface of the cell. Not greatly acidifying it but there is enough proton gradient there now you have a proton gradient directed from outside to inside) -So it meets the acidic microclimate, and free FA get protonated, because they are in acid. Once they are protonated they cannot stay in miscelle anymore.Miscelle cannot handle charges anymore so they have to leave the miscelle. -What is thought to happen is that they diffuse across the plasma membrane across the lipid bilayer into the enterocyte, as a free FA, because they are going to be lipid soluble. this really only applies to long chain FA (8 or more C). Short and median chain FA don't go through these process of being put in miscelle. They get digested it and diffused into the gut by themselves. -As the micelle progresses down the gut, it gradually gives off its cargo of fatty acids It is going to hit the surface and get protonated and gradually leave the miscelle -Probably the last thing that leaves the miscelle will be cholesterol. That is probably not going to be freely absorbed until you get down to the terminal ilium. -Cholesterol has specific transporters that pulls them from the apical solution -These miscelles gradually give up fatty acids cargo, and eventually they will just end up less and less fat there and become simple bile salt micelles and they just collapse. They disintegrate. -Free bile salt liberated. those bile salts are reabsorbed across the membrane at the terminal ileum and take back to the liver and re-processed

what is bilirubin?

Metabolism of Bilirubin a. Bilirubin is the breakdown product of hemoglobin. It's broken down by the SPLEEN. b. Liberated bilirubin binds to plasma albumin. c. Plasma albumin takes bilirubin to liver; which conjugates it to bilirubin+glucoronic acid to bilirubin glucuronide. i. This is excreted into bile duct. d. Bilirubin glucoronide is worked on by gut bacteria to bilirubin. e. This is processed to urobilinogen and excreted in fecal wastes. i. This gives color to feces. f. Some urobilinogen escape into systemic and back to liver. This gets back to systemic and is filtered by kidneys, which gives the color in urine. g. Bilirubin metabolism is important tool to diagnose jaundice.

Small Intestinal Motility

Modes: Slow waves MMC (motilin) Segmentation Peristalsis Functions: Basic electrical rhythm Sweeps intestine Mixing Net propulsion

signs and symptoms of HBV

Most children < 5 years old are asymptomatic. 30-50% of persons > 5 years of age have signs & symptoms. 90%of children will get chronic infection 5% in adults becomes chronic Symptoms when present can include: Constitutional: fever, fatigue & joint pain Gastrointestinal: loss of appetite, nausea & emesis Hyperbilirubinemia: jaundice, dark urine & clay-colored bowel movements Decompensated cirrhosis-Hepatic cirrhosis (compensated or decompensated) and hepatocellular carcinoma are complications associated with HBV and occur in about 30% of patients who develop chronic infection.

Alcoholic Liver Disease:

Most common liver disease in US: > 60% of chronic liver disease in most Western countries 40% to 50% of deaths due to cirrhosis 10-15% of heavy drinkers Treat with abstinence Chronic abuse responsible for up of 45% of hepatocellular carcinoma Wide range of severity Characteristics: Steatosis-accumulation of triglycerides within hepatocytes from moderate intake Alcoholic steatohepatitis (steatosis + hepatocyte injury) initially reversible ballooning degeneration, focal necrosis, neutrophils, Mallory-Denk Bodies, perivenular fibrosis Eventual progression to cirrhosis

what is the antidote for Tylenol toxicity?

N-acetylcysteine: Replenishes glutathione Binds NAPQI Restores sulfation conjugation Anti-inflammatory Anti-oxidant Vasodilating agent Oral or i.v. w/in 8-10 hr - 90% protection w/in 10-24 hr - 40-70% protection safe

what is the toxic intermediate? how is it metabolized?

NAPQI- is the toxic intermediate. At low doses its fine,but at higher concentration in the liver is overwhelmed (we do not have a lot of GSH) =toxicity a. APAP is the prodrug. It can be handled in many ways: i. Normally it can be glucoronidated using glucoronyl transferase and processed to glucuronyl derivated (95% total metabolites) ii. Some can get sulfated using sulfotransferase b. Issue comes: i. When too much Tylenol is used, the 2 pathways get saturated and you're forced to use a 3rd way. ii. The drug must pass down through CYP450 system and converted to *NAPQI 1. NAPQI is very toxic. 2. It transfers sulfohydryl groups to all proteins, which leads to crosslinking and cell death. iii. Normally, NAPQI is NOT present much in the liver to have a problem. iv. It can detoxify itself by reduction from NAPQI to glutathione (5% metabolites). c. Main point: i. Someone taking too much Tylenol has saturated their glucuronyl and sulfotransferase systems, which forces the drug to go through the CYP450 system and converts to NAPQI. ii. Only 5% of total acetaminophen will end up as NAPQI to glutathione process. If NAPQI is increased due to drug increased, then glutathione is used up and NAPQI build up→increase sulfate binding to proteins and ultimately acute liver failure.

in the basolateral membrane of the salivary cells there are ___ transporters

Na2+/K+-ATPase, a Na-K pump,and Na+/K+/Cl- cotransporter these work together to produce a high intracellular concentration of cl-

what is the difference in Na transport and Cl transport?

Na=absorption in villus Cl= secretion in crypt

Vitamin B12 Deficiency

Neurological symptoms Numbness, tingling, ataxia May progress to complete paraplegia Causes Dietary deficiency limited to strict vegetarians Autoimmune response to IF or parietal cells (pernicious anemia) Gastrectomy Resection or dysfunction of distal ileum

Adenocarcinoma of the Esophagus

Now over 50% of esophageal cancer in US Associated with Barrett's esophagus usually in distal esophagus Clinically: older patients, males>females, whites>blacks Similar presentation and prognosis to SCC adeno=glands carcioma=cancer

Where do the protons come from to make HCl?

One of the unique things about parietal cells is that they are stuffed full of this enzyme called Carbonic Anhydrase -Carbonic Anhydrase catalyzes the reaction between CO2 and H2O to give you protons and bicarbonate. -So protons from carbonic anhydrase, and bicarbonate. If all the protons are leaving out over the H/K/ATPase, we've got to do something with the bicarbonate. -So the basolateral membrane, we have a chloride bicarbonate exchanger. This lets us get rid of the bicarbonate and it brings in chloride so we've got chloride for this channel on the apical membrane that lets chloride out into the stomach.

what are the manifestations of a hepatocellular carcinoma?

Only 10% arise in otherwise normal livers Alcoholic cirrhosis Hepatitis B & C Non-alcoholic fatty liver disease Chemicals & toxins Hemochromatosis Wilson's Disease Glycogen storage diseases clinical features: -Painful, enlarging mass, abdominal discomfort Cachexia -Markedly elevated alpha-fetoprotein -Survival for a few months unless tumor is resectable or liver is transplanted -Soft, hemorrhagic tan masses -One large or many small tumors -Grows into vessels -Tumor cells resemble hepatocytes

describe phase 1 drug metabolism:

Phase I: Oxidation a. Key enzyme that's responsible for initial phase (oxidation) of drugs metabolism is cytochrome P450 (there are 40 know varieties). b. All are related and do stlightly different jobs. Cytochrome P-450 catalyzes the following reaction: a. Reaction: DH + NAD(P)H + H+ + O2 ⇒DOH + NAD(P)+ +H2O b. Drug (i.e Aspirin) is oxidized to salicylic acid. i. This process is IMPORTANT b/c a lot of drugs are prodrugs. ii. Prodrugs REQUIRE oxidation in order to become an active compound. iii. Sometimes, if you oxidize a drug, it will loose that activity (which is another problem). c. There are some things that we might take the can inhibit this phase I, which can decrease or increase the concentration of the oxidizing job. d. Example: i. Some drugs and environmental agents, (e.g. phenobarbital, cigarette smoke) also induce Phase I enzymes causing increased metabolism. ii. In contrast, some drugs, (e.g., cimetidine), inhibit Phase I enzymes, prolonging drug half-life.

describe phase 3 drug metabolism:

Phase III: Elimination process a. Conjugated drugs are excreted into bile and go through different transporters. b. These transporters are responsible for the secretion of contents out into bile. c. These transporters have broad specificities: take a lot of different anions. d. These are first pass effect. i. If you take a drug, it might have the go through all these metabolic step before you get your active compound. e. With intravenous drugs, you'll use MUCH LESS DOSE than pill b/c the pill has to go through first pass effect. i. Drug binding to albumin in the circulation also limits bioavailability.

what are the HAV immune globulin vaccines?

Pre-exposure: Travelers to intermediate & high HAV-endemic regions without enough time for HAV vaccine Post-exposure: (within 14 days) Household & other intimate contacts Selected situations in institutions & common source exposure -Two single-antigen vaccines are available (HAVRIX® & VAQTA®) A combination vaccine, TWINRIX®, contains both HAV & HBV antigens All are inactivated vaccines Schedule is 0 and 6 months Not recommended for children < 1 years old

what are the 2 ways we handle ammonia?

Proteins→urea→ammonia There are 2 ways: 1.Convert ammonia to ammonium (NH4) (converted using the colon). The same bacteria that are used are the same colon bacteria that are responsible for fermentation (non- absorbed carbs) and generation/breakdown of urea. -Because they are acidic, they can protonated to ammonium (which is NOT permeable). The ammonium is trapped in the colon and you'll excrete this in fecal waste. 2. Involved the liver. Some ammoniums are protonated, but some ammonia escape and get to the liver. Liver resynthesizes it into urea, and that urea will escape into systemic circulation where it gets filtered by the kidneys and excreted into the urine (75% of the time) However, some urea will make it back to the systemic circulation→colon→and go through the process again. -So b/t the liver and kidneys, you'll see short chain fatty acids. This allows blood ammonia level to be very low. -However, for someone w/o a functioning liver, they'll have elevated blood ammonia level. This is hepatic encephalopathy b. How do we treat this? Give LOTS of non-absorbed SUGAR. This allows for the usage of the physiology again. -Lots of sugars→convert to short chain FA→ excreted out as feces

what is small cell carcinoma esophagus?

SCC of the Esophagus Pathogenesis & Risk Factors: Lifestyle/Nutritional, Tobacco, Alcohol, Nitrosamines/nitrites, Nutritional deficiencies, Clinical, Hiatal hernia Chronic esophagitis Incidence is declining (adenocarcinoma of esophagus now more common; SSC used to be much more common than adenocarcinoma)

what is the sequence of events in a hepatocyte necrosis?

Sequence of events: 1Reactive metabolite(s) 2Lipid peroxidation/ROS 3Changes in ion homeostasis 4Mitochondria dysfunction 5Plasma membrane rupture Markers of cell death: Serum levels of hepatocyte enzymes Alanine aminotransferase (ALT) (cytosolic) normal=7-56 Aspartate aminotransferase (AST) (mitochondrial)=5-40

What Regulates Contractions?

Slow waves are an intrinsic property of the tissue BUT neural, hormonal, mechanical and chemical stimuli influence FREQUENCY and AMPLITUDE of contractions by increasing or decreasing the probability that a slow wave will reach threshold and/or fire spike potentials.

What slows gastric emptying?

Slows Emptying: Lipids Hypo or Hypertonicity Acid in duodenum Pressure in duodenum Distension in orad Distension in antrum Decreased contractions Large particle size Amino acids, small peptides Secretin, GIP

how do we absorb intact proteins?

So we can absorb proteins in two other ways, that's actually in whole intact proteins: -one of these is just simple phagocytosis. These cells (enterocytes) are always phagocytizing and pulling in from their apical membrane and replenishing it. When they do that they scoop up some of the luminal surface some material in the lumen, so when they do that they pull in some intact protein, just by chance and when they do that, most of those proteins are degraded as AA and delivered across basolateral membrane into the blood just like any other AA. - There are specialized cells in the gut called M cells (microfold) is to absorb INTACT proteins. They are part of immune system. The food you eat is considered an antigen. Their job is to take intake proteins present them to underlying macrophages and so the macrophages decide if they're going to mount an immune response or not to what you've eaten. -Very little of these proteins are degraded, most are delivered as intact proteins to underlying lymphocytes. This is part of the innate immunity!

signs and symptoms of HAV infection

Some persons, particularly young children, are asymptomatic. When symptoms are present, can include: Constitutional: fever, fatigue & joint pain Gastrointestinal: loss of appetite, nausea & emesis Hyperbilirubinemia: jaundice, dark urine & clay-colored bowel movements

A 57-year old man who have lived in New York city his entire life comes to the ER because of chest pain and shortness of breath. He is diagnosed with a massive pulmonary embolism and dies before receiving treatment. An autopsy confirms the diagnosis and also reveals a soft, yellow, greasy, grossly enlarged liver. Histological evaluation of the liver shows lipid accumulation within the hepatocytes, compressing and displacing the nucleus to the periphery of the hepatocyte. Which of the following is the most likely cause of these hepatic findings? A. Alcohol abuse B. Carbon tetrachloride exposure C. Diabetes mellitus D. Malnutrition E. Obesity

The correct answer is A. All of the conditions listed can cause steatosis of the liver in industrialized nations, however, the most common cause is alcohol abuse. The steatosis may take the form of either microsteatosis (multiple, small lipid vacuoles) or macrosteatosis ( a single large coalesced lipid vacuole in each cells); the two forms may also coexist. The triglyceride accumulation underlying the changes is related to alcohol induced damage to mitochondria and microsomal functions that normally degrade triglycerides.

A 32 year old man is on a high protein, low carbohydrate diet because he has heard that this will help him build muscle. Which of the following peptides needs to be released to increase the secretion of pancreatic enzymes into the small intestine so that he can digest these types of meals? A. CCK B. Gastrin C. Motilin D. Secretin E. Somatostatin

The correct answer is A. Release of CCK is stimulated by the presences of peptides, amino acids, or fatty acids in the small intestine. CCK acts on the pancreas to stimulate secretion of pancreatic enzymes that aid in the digestion of these compounds.

Which of the following is characteristic of saliva? A. Hypotonicity relative to plasma B. A lower HCO3- concentration than plasma C. The presence of proteases D. Secretion rate that is increased by vagotomy E. Modification by the salivary ductal cells involves the reabsorption of K+ and HCO3-

The correct answer is A. Saliva is characterized by hypotonicity and a high HCO3- concentration (relative to plasma) and by the presence of alpha amylase and lingual lipase. Because control of saliva production is parasympathetic, it is abolished by vagotomy. The high HCO3- concentration is achieved by secretion of HCO3- by the ductal cells.

All of the following are secreted by the pancreatic acinar cells EXCEPT: A. pepsin B. lipase C. amylase D. trypsin E. chymotrypsin

The correct answer is A. The presence of HCl converts the pepsinogen to pepsin in the stomach. Pepsinogen is secreted by chief cells in the stomach and HCl is released by the gastric parietal cells.

A 65- year old man presents to his family physician with a 3- month history of watery diarrhea. He is referred to a gastroenterologist who finds that the patient is also hypokalemic and achlorhydic and has an elevated serum level of VIP due to a pancreatic tumor (VIPoma). Which agent would be best to treat the patient's symptoms? A. gastrin B. octreotide C. glucagon D. bismouth subsalicylate E. suflasalazine

The correct answer is B. Ocreotide is used in the treatment of endocrine tumors such as gastrinomas, glucagonomas, and VIPomas to help alleviate the diarrhea. Bismuth subsalicyate is used to treat traveler's diarrhea and sulfasalzine is used to treat inflammatory diseases such as Crohn's disease. Gastrin is a GI hormone.

A 35-year old man is seen over multiple visits for complaints of ulcers, despite the use of ranitidine. Further studies find elevated levels of gastrin and evidence of ulcers involving the jejunum, suggesting a diagnosis of Zollinger-Ellinson syndrome. Which of the following agents would be the most useful in the management of this patient? A. Famotidin B. Omeprazole C. Misoprostol D. Propantheline E. Pepto Bismol

The correct answer is B. Omeprazole is an H-K-ATPase proton pump inhibitor useful in the treatment tot patients who have failed histamine H2 blocker therapy and patients with Zollinger-Ellison syndrome. Famotidine is a histamine H2-blocker. Misoprostol is used to prevent ulcers in patients taking non steroidal anti-inflammatory drugs (NSAIDS). Propoantheline is a cholinergic agent used in conjunction with other agents, rarely used alone. Ulcers associated with H. pylori can be treated with Pepto Bismol as part of the therapy.

A 64 yer old man undergoes a total gastric resection for adenocarcinoma of the stomach. He has dome well for 4 years but now presents with profound anemia, fatigue, and vague neurological complaints. Position and vibration sensation are markedly diminished and hyperreflexia is pronounced. Lab studies, including examination of bone marrow, reveal pancytopenia and other findings comparable with a megablastic anemia. He is likely suffering a deficiency from which essential vitamin? A. folate B. Vitamin B12 C. Vitamin C D. Vitamin D E. Vitamin K

The correct answer is B. The history of gastric resection is consistent with a deficiency of intrinsic factor which is required for absorption of vitamin B12 in the terminal ileum. Both folate and vitamin B12 lead to megaloblastic anemia, secondary to impaired DNA replication. In marked contrast to folate deficiency, vitamin B12 deficiency causes neurological dysfunction associated with damage to the lateral and dorsal spinal columns.

A 53 year old woman presents with a 6-month history of dysphagia, substernal pain, and melena. Her substernal pain is exacerbated when she eats larger meals or goes to bed for the evening. Her symptoms are due to dysfunction of which of the following sphincters? A. ileocaecal B. Lower Esophageaal C. Pyloric D. Sphincter of Oddi E. Upper esophageal

The correct answer is B. The patient has reflux esophagitis due to an incompetent lower esophageal sphincter. Her symptoms arise from the reflux of acid back into the esophageal mucosa. Her symptoms are exacberated by large meals, which lead to increased gastric secretion, and lying in bed, a position that allows more of the gastric contents to pass back through the incompetent lower esophageal sphincter.

A thin 32-year old man develops left hip pain and fever. A diagnosis of septic arthritis is established and he undergoes surgical drainage of the left hip joint. On the third day after surgery, it is noted that the fluid draining from the hip joint has becomes green and frothy, looking suspiciously like enteric contents. He is given charcoal granules by mouth and the granules are clearly visible in the hip drainage 6 hours later. In response to directed questions, he has a history of occasional episodes of colicky abdominal pain going back at least 10 years, sometimes accompanied by diarrhea. He never has had blood in his stools before. Which of the following is the most likely diagnosis? A. Appendicular abscess B. Chronic ulcerative colitis C. Crohn's disease D. Sigmoid diverticulitis E. Surgical damage to nearby intestinal loops

The correct answer is C. Fistulas between the GI tract and hip joints are very rare, but this man has developed one. Crohn's disease is notorious for its propensity to develop fistulas. These are typically within the abdomen, but they can form in unexpected locations like this one. His age and prior history are suggestive of this diagnosis. Chronic ulcerative colitis rarely fistulizes and often has bloody diarrhea. Sigmoid diverticulitis can fistulize but usually does so into the urinary bladder. it is preferentially a disease of older people. It has been described in young obsess men of Mexican descent, which this man does not fit. An appendices abscess could eventually point into the groin and migration of pus would occur on the right side and would produce a groin mass rather than a septic hip.

Which of the following conditions is caused by back flux of acid and a weak lower esophagus? A. Achalasia B. Mallory-Weiss Syndrome C. Hiatal hernia D. GERD

The correct answer is D. GERD is a chronic condition caused by acid reflux due to back flow from the stomach into the esophagus. The most frequent complication is a weak lower esophageal sphincter and risk factors include scleroderma and hiatal hernia. Symptoms would be heart burn, food regurgitation, hoarse voice, wheezing, and coughing. The condition is treated by antacids and proton pump inhibitors. Left untreated, GERD leads to Barrett's esophagus. Achalasia is caused by nerve dysfunction. It is characterized by a decreased propulsion of food down the esophagus. Mallory-Weiss syndrome is also caused by a weak lower esophageal sphincter but it is not due to acid reflux- it is due to mucosal lacerations from chronic vomiting. Hiatal hernia is caused by protrusion of part of the stomach through the diaphragm into the thoracic cavity.

Which of the following drugs would be best used to treat Crohn's disease in a 53 year old woman? A. Diphenoxylate B. Cimetidine C. Orlistat D. Sulfasalazine

The correct answer is D. Sulfasalazine is used to treat inflammatory diseases such as Crohn's disease. Diphhenoxylate is a morphine analogue used to treat diarrhea. Cimetidine is used to treat esophageal reflux. Orlistat is an agent used to manage obesity.

A 50 year old man with moderated familial hypertriglyceridemia is treated with gemfibrozil. Which of the following is the primary mechanism of action of this drug? A. Binding of bile acids in the intestine B. Inhibition of hepatic VLDL secretion C. Inhibition of HMG-CoA reductase D. Stimulation of HDL production E. Stimulation of lipoprotein lipase

The correct answer is E. Gemfibrozil works by increasing the activity of lipoprotein lipase, leading to increased clearance of VLDLs, which are elevated in familial hypertriglyceridemia.

Micelle formation is necessary for the intestinal absorption of: A. glycerol B. galactose C. leucine D. Bile acids E. Vitamin D F. Vitamin B12

The correct answer is E. Micelles provide a mechanism for solubilizing fat soluble nutrients in the aqueous solution of the intestinal lumen until the nutrients can be brought into contact with and absorbed by the intestinal cells. Because vitamin D is fat soluble, it is absorbed in the same way as other dietary lipids. Glycerol is a product of lipid digestion that is water soluble and is not included in micelles.

A patient with Zollinger-Ellison syndrome is expected to have which of the following changes? A. decreased serum gastrin levels B. Increased serum insulin levels C. Increased absorption of dietary lipids D. Decreased parietal cell mass E. Peptic ulcer disease

The correct answer is E. Zollinger-Ellison syndrome is a tumor of the non-beta cell pancreas. The tumor secretes gastrin which increases acid secretion. peptic ulcer, and parietal cell growth. Because the tumor does not affect beta cells, insulin levers are not affected. Absorption of lipids is decreased because increased H+ secretion decreases the pH of the intestinal lumen and activates pancreatic lipases.

how do you diagnose hep A?

The diagnosis is made by anti-HAV IgM Transmission is fecal-oral Incubation period is about 28 days (range 15-50 days) Rarely transmitted by blood to blood transfer Diagnosis is by anti-HAV IgM= youv have it for a couple of months IgG= you've had it for many years IMPORTANT= Total immune globulin (Total Ig) Combination of IgM & IgG Early infection - primarily IgM Late infection - primarily IgG

what are some of the drugs that can inhibit acid production

There are several drugs that you can use to inhibit acid secretion, or suppress it. -The most effective ones are the omeprazole family members, they work by binding to this H/K/ATPase and inhibiting its turnover. -Nexium is one of these drugs, its esomeprazole, it's in that family of drugs. "The Purple Pill" works by blocking the H/K/ATPase

how are oligopeptides absorbed?

These peptides are transported by PEPT1 transporter -PEPT1 - it's a little unusual. It doesn't use Na gradient, it uses H+ gradient At the apical memberane, another Na- H+ Exchanger (NHE) it moves H+ out of the cell and moves Na in, and its job is to maintain good intracellular pH. By transporting H out of the cell onto the apical surface, It sets up acidic microclimate. The area right about the brush border is slightly acidic (very slight gradient) but its sufficient to drive the uptake of oligopeptides using this transporter. -We absorb more AA as oligopeptides, once brought intracellularly, their broken down by intracellular proteases and then releases AA and the AA will exit the cell both the ones that come like this and the ones that come through PEPT1. They'll leave the cell by whole variety of different facilitated diffusive mechanism, some specific and some broad specificity. -There's a whole array of AA transporters on the basolateral membrane, there's a whole array on the apical membrane.

Vitamin Deficiencies

Thirteen vitamins necessary for health Fat soluble A, D, E, K Water Soluble B1, B2, Niacin, B6, B12, C, Folate, Pantothenic acid, Biotin

what is the purpose of the lipoprotein lipase

This enzyme will digest triglycerides into free fatty acids and allow penetration into different tissues for use/metabolism.

risk factors for HCV and prevention

Transfusion of blood or blood products < 1992 Intravenous drug use Hemodialysis High-risk sexual contact Children born to HCV + mothers Tattooing or body piercing in unlicensed facilities Occupational exposure to blood or blood products An organ, graft, or tissue transplant from an HCV+ donor Intranasal cocaine prevention: Prevent transmission from infected persons through percutaneous or mucosal contact with infectious blood Screening of blood, organ, & tissue donors Blood & body fluid precautions No vaccine available No benefit with immune globulin for prevention Keep cuts & skin lesions covered Potential for sexual transmission Potential for perinatal transmission no evidence to advise against pregnancy or breastfeeding Should NOT donate blood, organs, tissue, or semen Should NOT share household articles (e.g., toothbrushes, razors) NOT SEXUALLY TRANSMITTED

what is transit time of the large intestine?

Transit Time: Contents move at approximately 5-10 cm/h (max.)

the sodium-coupled solute transporter in absorptive diarrhea is malfunctioning and causes:

We are looking at problems with absorptive mechanisms. This is lactose intolerance. This transporter wouldn't be working and you'd lose the expression of SGLT-1 or the expression of lactase. You cannot absorb those nutrients and you get diarrhea. The same would be true if you lose the ability to absorb amino acids, but that's not common

thiamine B1 deficiency

Widely available in Western diet but lacking in refined foods Phosphorylated to active form during absorption in GI tract Involved in synthesis of ATP Cofactor in pentose phosphate pathway Maintains neural membranes In US, chronic alcoholics Developing countries, diet high in refined rice Deficiency targets peripehral nerves, heart, and brain Dry beriberi: Polyneuropathy due to myelin degeneration Begins in legs and moves to arms (toe drop, foot drop, wrist drop) Progressive sensory loss, muscle weakness and hyporeflexia -Wet beriberi: Heart disease due to peripheral vasodilation High output cardiac failure Eventual peripheral edema -Wernicke-Korsakoff syndrome: Hemorrhage and degenerative changes in the mamillary bodies of the brain Global confusion, ataxia, ophthalmoplegia, nystagmus, retrograde amnesia, inability to require new information

what primes the pancreatic secretion to start when you are before you eat? cephalic/gastric

You're going to get a small amount of gastrin released from these G cells and that's going to bind to CCK-A receptors and give you a little bit of pancreatic enzyme release -So you start getting ready for this meal that is about to arrive: you'll get some pancreatic enzyme release, vagal input of acinar cells so you get Ach, stimulating pancreatic enzyme release - you'll get a little bit of fluid -The vagus will have muscarinic receptors for Ach on the duct cells so you get some pancreatic fluid being made and enough to wash a little bit of enzyme out into the small intestine

zinc Mineral Deficiencies

Zinc Distincive rash around eyes, nose, mouth, anus (acrodermatitis enteropathica) Growth retardation Impaired wound heling Infertility Impaired night vision (altered vit. A metabolism)

what is the difference in zonal distribution and focal necrosis?

Zonal distribution (i.e. Zone 3 initially affected in ischemic conditions) Focal necrosis (single cell death), bridging necrosis , submassive necrosis (entire lobule) or massive (most of liver)

what are the 3 zones of the liver?

Zones of liver: Zone 1 - cells around portal vein synthesize glycogen, plasma proteins and responsible for urea/NH3 handling; can be damaged by ingested toxins [O2] is high; Zone 2 - cells between zone 1 and 3 Mid-zonal Zone 3 - cells around hepatic vein; responsible for drug detoxification, glycolysis,lipogenesis [O2] is poor; cells vulnerable to hypoxia, ischemia

Most of our phase 1 reaction(s) occur in ZONE ___.

a. Most of our phase 1 reaction(s) occur in ZONE 3

why is acetaminophen toxic?

a. Why it's problematic? i. B/c of the way it's processed in the liver. b. APAP→active component of Tylenol. c. Popular OTC analgesic and antipyretic d. Does not cause gastric irritation e. Therapeutic dose is safe (1-4 g/day) f. Narrow therapeutic window g. Overdose (> 10-15 g) can produce hepatic necrosis that can be fatal h. Acute hepatocellular injury in zone 3 i. Therapeutic doses can also be fatal j. (~4-6 g/day chronically) k. Age, Gender l. Chronic alcohol consumption m. Drug:drug interactions n. Fasting o. Genetics a. Take overdose of acetaminophen (see cell death). There's widespread cell death

-If you lose fluid _ the level of the pylorus, you will shift into a metabolic alkalosis _____ the pylorus is acidosis.

above= alkalosis below=acidosis

in the GI tract: absorption occurs at the ___and secretion occurs at the____

absorption=villus secretion=crypt

what are the 4 most important things in gastric secretion?

acid, pepsinogen, mucous, IF

the ____ are responsible for the source of the vast majority of enzymes in pancreas. Those enzymes get stored in _____. The pancreas has a very high rate of de novo protein synthesis.

acinar cells zymogen granules

bile acids are reabsorbed from the blood and secreted into the bile canaliculi of the liver via:

active transport

how an you tell the difference in acute and chronic inflammation?

acute= neutrophils chronic= neutrophils are gone and you see lymphocytes

what is the course of chronic HCV infection?

after 1 week HCV RNA high, anti-HCV high, and ALT alternates

what is the course of acute HCV resolution?

after 6 weeks you get anti-HCV

what are the active enzymes that the pancreas secretes?

alpha- amylase, carboxyl ester lipase, lipase, RNase, DNase

what is the major protein in saliva?

amylase

what is the main enzyme used to digest starch?

amylase it is in the pancreas and the saliva

folate deficiency

associated with poor diet Medication (phenytoin, methotrexate) No neurologic symptoms

how does secretion from the salivary duct work?

at the Chloride/Bicarb exchanger= In this duct cell, it's seeing lots of NaCl. - NaCl is then seeing a Na-H exchanger, and the Cl-HCO3 exchanger. Chloride is taken up by this exchanger into the duct cell in exchange for bicarbonate, so that's where the bicarbonate component of saliva comes from. -At the same time, it is swapping sodium for protons. So sodium is taken up across the exchanger, in exchange for protons. -Now we have bicarbonate and protons out here. The protons are being reabsorbed by this exchanger, a K-H exchanger which is thought to exist here. -left with a lot of bicarb sitting out here in the lumen all of these transporters are rate limited - there is the ENac (its presence goes up as you become more dehydrated, it is controlled by aldosterone)

what all occurs in zone 1?

b. Zone 1 (PP hepatocytes) - oxidative energy metabolism - beta oxidation, amino acid metabolism, ureagenesis, gluconeogenesis, cholesterol synthesis, and bile formation.

-What are the electrolyte changes in balance? In vomiting, you increased loss of protons. In diarrhea, you see loss of __

because bicarb is secreted into SI and colon. You lose a lot with diarrhea. You develop a metabolic acidosis. You can get an increase in blood chloride. You have that sodium-bicarb exchanger. -You get the same contraction of volume in the extracellular fluid and you can die from hypervolemia from diarrhea just as you can from excessive vomiting.

ammonia is exceptionally permeable:

becuase it is lipid soluble. it can cause hepatic encephalopathy problems in alcoholics. BUT ammonium is NOT lipid soluble and is trapped

what is the exocrine secretion of the liver?

bile

how are bile acids absorbed from the blood in the gut?

bile acids are taken from the blood via the Na cotransporter by secondary active transport into the hepatocyte. Once it gets into the hepatocyte, the hepatocyte might reconjugate it or modify it with a different group on the molecule (sometimes they get a sulfate added, which makes them a tertiary bile acid) then into the the bile canaliculus in a channel via primary active transport -This is referred to as the enterohepatic circulation of bile

what is the problem with bile acids and lipase? how is it solved?

bile acids bind to fat droplets and displace lipase from the surface of the droplet. - an enzyme called colipase which is secreted from the pancreas. it is the CRITICAL intermediate between the two

most of the acid in the stomach is secreted in the ___. the fundus secretes___ and ___.

body=acid, IF, pepsinogens, lipase fundus and pyloris=mucous and bicarb

what does urease do?

breaks urea down into Co2 and ammonia

where is enterokinase located?

brush border of duodenum

-Once digestion with a-amylase has occurred in the lumen of the gut, the digestion of these disaccharides or limit dextrins actually occur at the ____.

brush borders The enzymes that digest the disaccharides are bound at brush borders (this is called contact digestion), because the sugars have to go to the brush border and contact the brush border to be digested

what all occurs in zone 3?

c. Zone 3 - glycogen synthesis, glycolysis, lipogenesis, ketogenesis, xenobiotic metabolism, and glutamine formation.

Ach, epi and substance P trigger___

calcium which causes fluid to be released

what is the main enzyme in regulating urea synthesis and is heavily expressed in the hepatocytes in zone 1 and 2

carbamoylphosphate synthase

what are the exopeptidasea?

carboxypeptidase A and B these work at the terminal ends

What are tumors of the gallbladder?

carcinoma of the glallbladder: Very uncommon F>M Gallstones often present Manifestations similar to cholelithiasis < 5% five year survival Cholangiocarcinoma: Adenocarcinoma arising from intrahepatic or extrahepatic bile ducts

what secretes pepsinogen?

chief cells in the oxynitic gland and mucous cells of the pyloric glands

what are bile acids derived from?

cholesterol

what is the last thing left in the miscelle to be absorbed?

cholesterol

We also have one enzyme that will work on cholesterol is called ___

cholesterolester hydrolase

adenocarcinoma of the stomach is __in US

declining for 6 decades More common in men 20% 5-year survival

there are primary and secondary bile acids. what causes the change?

digestion via normal intestinal bacteria The primary bile acids are the ones made in the liver, while the secondary bile acids are the ones made in the gut by the bacteria (by removing the 7-OH group) -Tertiary bile acids are made in the liver

during the gastric phase, the neural regulation of pancreatic secretion is by:

distension induces vagovagal reflex from stomach wall to pancreas

pancreatic cells have lots of__

endoplasmic reticulum because they have a high rate of protein synthesis. they are continually making digestive enzymes

what converts trypsinogen to trypsin?

enterokinase

once the fatty acids bind with the FABP in the smooth ER, they combine with a protein called___

enzyme coA enzyme coA+ ATP + FA= acyl coA

what is a carotenoid?

fat soluble vitamin A, D,E,K

hepatitis A

fecal-oral transmission NO chronicity prevention: handwashing, clean water, immune globulin and immunization

what is Hemocromotosis

high amount of iron with have diabetes, arthritis, tanned skin, liver problems because the iron will precipitate into these organs -We have very specialized mechanism with dealing with iron because iron is surprisingly very toxic. -Although of course we need it. You have to have iron if you are going to make hemoglobin. -We need iron for cofactors for many other different enzymes. -But it is actually very toxic and there are lots of cases in the emergency room, where little kids are rushed into the hospital because they have ingested mom's iron pills and they are slowly being... generated rather quickly, being poisoned by having too much iron in the blood -They have very lasting neurological effect in kids. It makes them really hyper and maybe they crash rapidly and become very comatose

primary bile acids are more _____ than secondary

hydrophilic, but they both are amphipathic

when the body is getting ready to vomit what occurs?

hypersalivation, GLOTTIS CLOSES, diaphragm contracts and moves out of the way, soft palate rises, strong contractions of the abdominal muscles, reverse peristalsis

saliva is __tonic

hypotonic

the liver stores glucose as :

i. Between meals and when insulin levels are low and glucagon levels are high, the liver serves as a source of plasma glucose, both by synthesizing glucose and by generating it from the breakdown of glycogen.

There are 3 stages to drug metabolism:

i. First→oxidation (reduction, hydrolysis) ii. Second→conjugation iii. Third→elimination

where is albumin synthesized?

in the liver

where is intrinsic factor made?

in the parietal cells it is the most important secretion of the gastric mucosa because if you don't absorb B12 you develop pernicious anemia

glycogen synthesis occurs:

in zone 3

what is emulsification?

it happens in the stomach, there is a little salivary emulsification, but not much. the gastric lipase works at pH of around 4. as the stomach moves the food around it begins to make fat droplets. -fat droplets become coated with phospholipids because they are amphipathic which prevents the fat droplet from coalescing.

what is FABP?

it is a fatty acid binding protein present in the sooth reticulum of the cell used to resynthesize fatty acids in the cell

the chemoreceptor zone is important becuase

it is the trigger zone for the vomit reflex

what are the products of lipase digestion?

it liberates free fatty acids from the glycerol backbone

bile acids are energy expensive to make and require many materials how does the body combat this?

it recycles them - they are made in the liver, are secreted into the bile ducts, spend some time in the gallbladder, get excreted into the duodenum, and then are reabsorbed at the terminal ileum. They then enter the portal system and are returned to the liver.

what is pepsin and how is it made?

its is made in chief cells as pepsinogen and cleaved by acid in the stomach to make pepsin. it is an endopepsidase- it chops up polypeptides anywhere along a polypeptide chain

fat soluble vitamins are absorbed in the:___

jejunum

a. Most fat digestion is done in the end of ____

jejunum. It doesn't completely finished until it gets to the end of ileum

where are calcium and iron absorbed?

jejunum. but starts in the stomach with HCl secretion promotes solubilization of ca2+ and Fe3+ and release of food bound vitamin b12 (requires intrinsic factor)

what ions dose the saliva usually contain?

k, na, bicarbonate, cl

what does CCK do?

know 3 things that CCK does - pancreatic enzyme secretion, contraction of the gallbladder, and relaxation of the sphincter of Oddi -Ach also helps cause the smooth muscle to contract

In the gallbladder the tight junctions are relatively leaky, the consequences for that is that for small movements of _ you can move a lot of ___.

leaky: small salt= lots of water movement tight: you have to move a lot of salt to move water because they are not very permeable. -Tight junctions can be made more tight or more loose in response to different hormones or different circumstances going on. Some tight junctions can be regulated in response to stimulation.

Secondary bile salts are more ____, so they can more easily diffuse across the cell membrane of the enterocyte and get taken up into the blood -Primary bile acids are not as ___ and require a transporter

lipophilic

glucose-6- phosphatase is only found in the

liver

the largest organ in the body besides the skin is the____

liver it is 2-5% body weight in adult

what is a malabsorptive diarrhea?

osmotic diarrhea -A malabsorptive diarrhea is something that happens when you cant absorb nutrients. -When that solute co-transport that we have talked about as being so important for reabsorbing water - when that isn't working, you have malabsortive diarrhea. Some places you might see that is lactose intolerance. Lots of people are lactose intolerant. Most animals lose the ability to digest milk sugars after weening. As people get older, they tend to lose the ability to digest those sugars - the lactose - the milk sugar well. Because you cant digest that lactose, you cant break it apart, you end up with an increased osmotic load in the gut and you end up with diarrhea. -Also, you might have malabsportive diarrhea if you lose epithelial surface. Remember we talked about how the gut is all about surface area - contact of surface area with the contents. There are things that will cause you to lose surface area. An example is celiacs disease. That is an allergy to gluten. You lose the villi because of an autoimmune attack.There is distruction in the villi in people who eat gluten. If those people avoid gluten, then the villi will grow back. But they have severe malabsorptive diarrheas from loss of surface area. You get increased osmotic load and increased diarrhea. -Some viruses will do the same thing. There is the flattening of villi virus leading to malabsportive diarrhea. -You can also get it from increased intestinal mobility. If material is moving through your gut fast, then you don't have sufficient time for those nutrients to be absorbed leading to malabsorptive diarrhea. caffeine? -People who have a small bowel bacteria overgrowth can see that. The bacteria ferment the nutrients, you get swelling in the bowel and that will decrease intestinal transit time.

in gluconeogensis all precursors must be converted into__

oxaloacetate

what are the 3 amino acids that are good at stimulating release of CCK?

phenylalanine, methionine, and tryptophan (F, M, W)

enzyme that works specifically on phospholipids, such as lethicin is called___

phospholipase A2

the difference in primary and secondary hematochromatosis:

primary: autosomal recessive Presents in 5th decade Males > females (due to menstrual blood loss) secondary= rare

what is the first step in protein digestion?

proteases -pepsin in stomach

what is the role of bile acids?

required in the formation of miscelles. they are really strong detergents this forms a true solution not just an emulsification

what are the lobes of the liver?

right, left, caudate, and quadrate lobes

what is the major difference in salivary and pancreatic ducts?

salivary are impermeable to water while pancreatic are very permeable to water

what is fibrosis?

scarring

We know that _____ produces water and bicarb from the bile ducts just like it does in pancreatic ducts

secretin

how do we absorb the monosaccharides glucose and galactose once we have digested them?

the key transporter is the SGLT1 (sodium glucose linked transporter 1) via secondary active transport -On the basolateral membrane you have Na-K Atpase, setting up an inwardly Na directed gradient. That encourages Na to move down its gradient from the lumen of the gut to the enterocyte. When it does that, and it uses the protein, it will take a glucose or galactose molecule with it, and it doesn't care if it uses glucose or galactose. They both have equal affinity for that transporter. So this is secondary transport because it exploits the gradient that's set up by the Na pump (active transport system). This is therefore referred to as secondary active transport. -Once the sugars get into the cell there's a passive transporter or the basolateral membrane: GLUT2 which allows sugar to difuse out of cell and into the blood. -Remember that this is secondary active transport - relying on the Na-K atpase to set up an inwardly directed gradient.

what is the most important job of CCK?what are its other jobs?

the secretion of pancreatic enzymes -CCK has several jobs, and one of the most important ones is the secretion of pancreatic enzymes. This is a true authentic action of CCK - it binds slow CCK-A receptors -In the pancreatic acinar cell, you will find some receptors for secretin and some receptors for VIP - this will give you a little bit of pancreatic enzyme secretion but not much. -The most important hormone for pancreatic enzyme release is CCK. -Gastrin - this will bind to CCK-A receptors; it doesn't bind very well but you'll get some pancreatic enzyme release b/c gastrin will bind CCK-A receptors. -We have standard protein coupling pathways here: adenylate cyclase activating protein kinase A which will cause enzyme release, and here we have calcium signaling cascade that will give fusion of secretory granules and enzyme release

what is the main job of the colon?

to absorb water and dehydrate waste, but most of the water is reabsorbed in the small intestine

what is the job of the gallbladder?

to concentrate bile

most common secretory diarrheas are

traveler's diarrhea, cholera, salmonella, and c. diff.

what is the composition of chylomicrons?

triglyceride=90% cholesterol ester=1-3% cholesterol=1% protein=1.5% phospholipid=6.5% traces= vitamins and xenobiotics

what is the difference between the crohn's and ulcerative colitis?

ulcerative has an Absence of granulomata, Continuous lesions, begins in rectum and spreads proximally, Usually limited to mucosa/submucosa rare fibrosis, serosal involvement, fistula formation crohn's= transmural, skip lesions, can be in any part of gi tract

mucous secretion is under the control of

vagal efferents and prostaglandins the same things that control acid secretion also control the secretion of mucous and bicarbonate

during the cephalic phase, the neural regulation of pancreatic secretion is by:

vagal efferents input into acinar cells and ducts, accounts for 10% secretion


Conjuntos de estudio relacionados

Final Test for MPhil - Comparative Government

View Set

CHAPTER 17 - OLDER ADULTHOOD: PERSONALITY AND SOCIOCULTURAL DEVELOPMENT

View Set

Chapter 7: Memory: Constructing and Reconstructing Our Pasts

View Set

Management of Patients with Oncologic Disorders (Chapter 15)

View Set

Business Law II - Chapter 10 6th ed

View Set

int quiz & Expressions quiz (C for Everyone: Programming Fundamentals - Week 2 Coursera)

View Set

Spinal exam 1 Practice Questions

View Set